MBE Questions Flashcards

1
Q

Civ Pro - Post Trial Matters

A three-car accident occurred in which the drivers were a citizen of State A, a citizen of State B, and a citizen of State C. The State A citizen filed a negligence action against the other two in federal district court and lost his case.

After judgment, may the State C citizen assert and maintain a negligence action against the State B citizen seeking damages for the injuries the State C citizen sustained in the same accident?

A

Yes, because, while the State C citizen could have asserted the claim as a cross-claim in the prior action, he may wait and assert it as an independent action.

  • The State C citizen may assert and maintain a negligence action against the State B citizen. The claim could have been asserted as a cross-claim in the prior action because it arose from the same transaction or occurrence, but cross-claims are never compulsory.
How well did you know this?
1
Not at all
2
3
4
5
Perfectly
2
Q

Civ Pro - Trial Matters

A truck collided with a car in State A, injuring the driver of the car. The driver of the car filed a civil action in federal district court in State A against the trucking company to recover damages for the driver’s injuries. The trucking company filed a motion to dismiss the action against it on the grounds that the court in State A did not have personal jurisdiction over it. Although the driver of the truck was the company’s employee, the trucking company argued that the driver did not have authorization to drive the truck to State A. Following a hearing, however, the court ruled that the trucking company was subject to the court’s personal jurisdiction.

Another two months passed, and the trucking company did not file an answer. The driver of the car then filed a motion asking that the clerk of court make an entry of default, and the clerk did so.

What procedure must the driver follow to obtain a default judgment against the trucking company?

A

File a motion to have the judge enter the default judgment, which the judge may do as long as the trucking company receives additional notice of the motion for default judgment.

  • The driver must file a motion to have the judge enter the default judgment. A defendant against whom a default is entered loses the right to contest liability unless the entry of default is set aside. However, the amount of damages must still be determined before a default judgment may be entered, and the defaulting party can be heard at the hearing for damages. If the defendant has “appeared,” even though he has not answered, he must be notified of the request for a default judgment by first-class mail at least seven days before the hearing on the application for a default judgment. Appearance includes any actual formal appearance before the court and any other action that clearly indicates that the defendant intends to contest the case on the merits. Here, the trucking company made an appearance when it challenged the court’s jurisdiction.
How well did you know this?
1
Not at all
2
3
4
5
Perfectly
3
Q

Civ Pro - Trial Matters

A plaintiff sued a trucking company for negligence in federal court, alleging that its employee, a driver for the company, was acting within the scope of his employment when driving the truck that hit her. The company’s first response to the complaint was to file a motion for summary judgment, together with an affidavit, alleging that the driver was not acting within the scope of his employment when the driver hit her, because the accident happened in State A and the driver was only authorized to be in State B at the time. Under applicable law, the company would not be liable if the driver was not acting within the scope of his employment. The plaintiff responded with a competing affidavit from an eyewitness who saw the driver texting immediately before the crash.

How should the court rule on the company’s motion?

A

Grant the motion, because there is no dispute as to a material fact.

  • The court should grant the motion. Summary judgment should be granted in cases such as this, where there is no genuine dispute of material fact such that the moving party (the trucking company here) is entitled to judgment as a matter of law. Here, the trucking company established through an affidavit that the driver was not acting within the scope of employment (by driving in an unauthorized state), which would negate the company’s liability. Affidavits may be presented in support of a summary judgment motion if they are made on personal knowledge by someone competent to testify and the facts would be admissible in evidence. There is nothing here to show any challenge to the affidavit. To the contrary, the plaintiff’s affidavit fails to address the company’s motion, so the court should consider the facts in the company’s affidavit (that the driver was not within the scope of employment) undisputed for the purpose of granting summary judgment. A motion for summary judgment is appropriate at any time until 30 days after close of all discovery; as a result, the motion here is timely and not premature.
How well did you know this?
1
Not at all
2
3
4
5
Perfectly
4
Q

Civ Pro - Pretrial Matters

A tourist from State A was severely injured in a bar fight in State B. The tourist filed a battery action against one of the bar’s patrons, seeking $100,000 for his injuries. The defendant patron claims that the tourist is mistaken about who hit him. The patron says that he did not touch the tourist. The patron claims that it was the bar’s bouncer-who looks like the patron-who hit the tourist and then continued to pummel him. The bouncer claims that he never touched or harmed the tourist.

Can the patron assert a third-party claim against the bouncer to bring him into the action?

A

No, because the patron has no legal basis to assert a claim against the bouncer and is not seeking to recover from the bouncer any portion of the patron’s liability to the tourist.

  • The patron cannot assert a third-party claim against the bouncer to bring him into the action. Third-party claims may be asserted only to obtain recovery, and that recovery must be for the defendant/third-party plaintiff’s own liability to the plaintiff. Since the patron is not seeking to recover from the bouncer any portion of the patron’s liability to the tourist, he cannot assert a third-party claim.
How well did you know this?
1
Not at all
2
3
4
5
Perfectly
5
Q

Civ Pro - Pretrial Matters

A developer contracted with a general contractor to build an office building, and completion of the building was two years late. The developer filed a breach of contract action in federal district court against the general contractor, seeking damages caused by the delay. The general contractor filed a third-party claim against a major subcontractor, claiming that the subcontractor caused any delay and should be liable to the general contractor for anything the general contractor has to pay the developer. The subcontractor believes that the developer interfered with the subcontract and that the developer’s interference caused not only the delay but also substantial cost overruns for the subcontractor.

May the subcontractor assert a claim in the pending action against the developer seeking payment for the cost overruns?

A

Yes, because the subcontractor’s claim against the developer arises from the same transaction or occurrence as the developer’s original claim, but the subcontractor may assert the claim in an independent action if it prefers.

  • The subcontractor may assert a claim against the developer in the pending action. A third-party defendant may assert a claim against the plaintiff if the claim arises out of the same transaction or occurrence as the plaintiff’s original claim. (A) is therefore incorrect. Because the subcontractor’s claim seeking payment for cost overruns and the developer’s original contract claim arise out of the same transaction or occurrence, the subcontractor may assert its claim against the developer. (B) is incorrect because the subcontractor’s claim does not have to seek indemnity; it must arise out of the same transaction or occurrence as the original claim. (C) is incorrect because a third-party defendant’s claim against the plaintiff is not compulsory. Therefore, failure to assert the claim in the pending action would not bar the subcontractor from asserting it in an independent action.
How well did you know this?
1
Not at all
2
3
4
5
Perfectly
6
Q

Civ Pro - SMJ

An employee who was fired plans to sue her former employer, claiming that the employer is liable for both wrongful termination under state law as well as violation of federal employment discrimination statutes, claiming total damages in the amount of $50,000. The employer and employee are citizens of the same state.

A

Yes, because the federal court has federal question jurisdiction over the federal statutory claim and supplemental jurisdiction over the state law claim.

  • The employee may assert these claims together. Federal question jurisdiction is available when the plaintiff, in his well-pleaded complaint, alleges a claim that arises under federal law. Here, the federal employment discrimination claim arises under federal law. Thus, a federal court has federal question jurisdiction over the claim. When the federal court has subject matter jurisdiction over one claim, it has discretion to exercise supplemental jurisdiction over related claims that derive from the same common nucleus of fact and are such that a plaintiff would ordinarily be expected to try them in a single judicial proceeding. (Essentially, this means that the supplemental claim must arise from the same transaction or occurrence as the claim invoking federal subject matter jurisdiction.) In the instant case, the federal employment discrimination claim is very closely related to the state wrongful termination claim. Each will deal with much of the same factual issues, and it would be expected for a plaintiff to bring such claims in a single action if at all possible. Thus, the court has supplemental jurisdiction over the state wrongful termination claims.
How well did you know this?
1
Not at all
2
3
4
5
Perfectly
7
Q

An employee filed an employment discrimination action against her employer in federal district court. The employee alleges that she has not been promoted because of her gender. She intends to call a co-worker as a witness at trial. The co-worker will testify that a senior manager of the employer told the co-worker that the employee would not be promoted because the employer deemed women to be poor managers.

Is the identity of the co-worker subject to discovery by the employer?

A

Yes, the employee must disclose the identity of the co-worker even without any request from the employer.

How well did you know this?
1
Not at all
2
3
4
5
Perfectly
8
Q

Yes, the employee must disclose the identity of the co-worker even without any request from the employer.

A

The court should deny the venue motion, because the defendants waived any objection to venue when they did not raise the objection in their pre-answer motion objecting to subject matter jurisdiction.

*The court should deny the venue motion. Objections to venue are waived if not asserted in the defendants’ first response to the complaint-whether that first response is the answer or a Rule 12(b) pre-answer motion. Here, the defendants did not object to venue in their first pre-answer motion; they only objected to subject matter jurisdiction.

How well did you know this?
1
Not at all
2
3
4
5
Perfectly
9
Q

Civ Pro - Jurisdiction/Venue

A plaintiff filed an action against a defendant in federal district court in State A, and properly served process on the defendant at his residence in State B. The defendant timely filed an answer denying the material allegations and also moving to dismiss the complaint on the ground that the court lacked personal jurisdiction over him.

Is the defendant’s objection to personal jurisdiction timely and proper?

A

Yes, because the defendant objected to personal jurisdiction in his first response.

  • The objection was timely and proper. Federal Rule of Civil Procedure 12 provides that objections to personal jurisdiction may be asserted by pre-answer motion or in the answer-provided that the objection is asserted in the defendant’s first response. If the objection to personal jurisdiction is not presented in the first response, the objection is waived.
How well did you know this?
1
Not at all
2
3
4
5
Perfectly
10
Q

Civ Pro - Pretrial Matters

A homeowner discovered that the siding on his house was defective and had allowed water to enter the structure, causing damage to the wood framing. The homeowner tried for some time to negotiate a settlement with the corporation that the homeowner believed had manufactured the defective siding. When no settlement was forthcoming, the homeowner filed an action in federal district court against the corporation one week before the statute of limitations expired. Service of process was effected on the corporation several months later. After inspecting the home, the corporation filed and served its answer in which it denied manufacturing the siding used on the homeowner’s house. Upon examining the corporation’s evidence, the homeowner conceded that the siding was manufactured by another company. With leave of the court, the homeowner then filed an amended complaint substituting the actual manufacturer of the siding for the original incorrect defendant. The amended complaint was served on the manufacturer approximately seven months after the original complaint was filed and after the statute of limitations had expired. The manufacturer was unaware of the action until it was served with the amended complaint. The manufacturer filed a motion for summary judgment on the grounds that the homeowner’s claim against it is barred by the statute of limitations.

How should the court rule on the motion?

A

Grant the motion, because the amended complaint was filed after the statute of limitations expired and the actual manufacturer did not receive timely notice of the action.

  • The court should grant the motion for summary judgment on the basis that the claim is barred by the statute of limitations. Amendments substituting a new defendant for one originally named are allowed and the amendment will relate back to the time the original complaint was filed if (i) the claims in the amendment arise from the same transaction or occurrence as the claims set out in the original pleading; and (ii) within the time allotted for serving the original complaint (90 days from filing per Federal Rule 4(m)), the new defendant received such notice of the action that it will not be prejudiced in defending on the merits and knew or should have known that the action would have been against it (the new defendant). Here, the manufacturer received no notice of the action within the time allotted for service and could not have known about the plaintiff’s mistake in naming the defendant. The amendment thus does not relate back and is time-barred.
How well did you know this?
1
Not at all
2
3
4
5
Perfectly
11
Q

Civ Pro - Post Trial Matters
A group of employees brought a class action lawsuit in federal court against their employer, alleging employment discrimination and the violation of several labor laws. The court granted certification of the class action. Wanting to force each employee to litigate individually, the employer immediately files an appeal.

Can the court of appeals hear this appeal?

A

Yes, because an order granting certification of a class action is immediately appealable.

The court of appeals can hear this appeal because a district court’s order granting or denying certification of a class action can be appealed within 14 days of entry of the order.

How well did you know this?
1
Not at all
2
3
4
5
Perfectly
12
Q

Civ Pro - Trial Matters

A car owner sued a crane operator for negligence in a federal court because the crane operator backed over and crushed her car. During jury selection, the car owner’s lawyer wished to exercise one of her peremptory challenges to excuse some potential jurors.

Which of the following reasons would provide the best grounds for the crane operator’s lawyer to object to the peremptory challenge?

A: Excusing the only potential male juror because he is a crane operator, just like the defendant.
B: Excusing a second African-American juror.
C: Excusing the juror with multiple tattoos.
D: Excusing the juror who has been unemployed for two years.

A

B: Excusing a second African-American juror.

  • Peremptory challenges allow an attorney to disqualify a potential juror because the juror displays an attitude or some characteristic that appears unfavorable to the attorney’s client but that does not rise to the level of bias that would present grounds for a challenge for cause. A party cannot use peremptory challenges if the court suspects the challenge is for race, national origin, religion or gender, which violate the juror’s equal protection rights under the Fourteenth Amendment. If the court detects a pattern of excluding jurors for any of these reasons, the opposing party can object, and the party may be required to justify the challenge by providing other nonobjectionable means. Here, exercising a peremptory challenge to excuse a second African-American juror from the panel without any other reason appears that it may be based on race. Therefore, choice (B) provides the best choice for the defense to object to the plaintiff’s peremptory challenge.
How well did you know this?
1
Not at all
2
3
4
5
Perfectly
13
Q

At a federal trial for breach of contract, the plaintiff presented evidence of both contractual and consequential damages amounting to $115,000. The jury, however, awarded the plaintiff $250,000 in damages. The defendant filed a motion for a new trial, arguing that the damage award was clearly excessive.

If the court agrees that the award is excessive, the court may:

A: Deny the motion for a new trial but lower the damage award and require the plaintiff to accept the remittitur.
B: Deny the motion for a new trial contingent on the plaintiff accepting a remittitur.
C: Grant the motion for a new trial only because the jury award may not be changed.
D: Grant the motion for a new trial only because remittitur is not allowed in federal court.

A

B: Deny the motion for a new trial contingent on the plaintiff accepting a remittitur.

  • The court may deny the motion contingent on the plaintiff accepting a lesser award by means of remittitur. If the judge believes that the jury’s compensatory damages award is so excessive as to “shock the conscience” (or, in a diversity case, if the award meets the state standard for excessiveness), the judge may order a new trial or may offer the alternative of remittitur. When offered remittitur, the plaintiff is given the choice between accepting an award less than that awarded by the jury or submitting to a new trial.
How well did you know this?
1
Not at all
2
3
4
5
Perfectly
14
Q

Civ Pro - SMJ

A driver, domiciled in State B, struck a car driven by a father domiciled in State A and his young daughter, who was living with her mother in State B. The father commenced an action in federal court against the driver in his own right and on behalf of his daughter, seeking $80,000 for his injuries, which were severe, and $5,000 for his daughter’s injuries, which were minor.

Does the court have subject matter jurisdiction of the claim for the daughter’s injuries?

A

No, because the daughter is a citizen of State B.

  • There is no subject matter jurisdiction for the daughter’s claim because both she and the defendant driver are citizens of State B. Supplemental jurisdiction is not available here because supplemental jurisdiction cannot be used to override the complete diversity rule.
How well did you know this?
1
Not at all
2
3
4
5
Perfectly
15
Q

Civ Pro - PJ/Venue

A resident of the District of State A properly brought a diversity action in federal court against a resident of the District of State B and a resident of the District of State C for a cause of action that arose from events that occurred in the District of State D.

In which judicial districts is venue proper?

A: Only in the District of State B and in the District of State C.
B: Only in the District of State D.
C: Only in the District of State A.
D: In the District of State B, the District of State C, and the District of State D.

A

B: Only in the District of State D.

  • Venue is proper only in the District of State D, where the events giving rise to the claim occurred.
How well did you know this?
1
Not at all
2
3
4
5
Perfectly
16
Q

Civ Pro - Pretrial Matters

A motorist from State A struck and injured a pedestrian in State B. The pedestrian, a State B resident, brought an action in a State B federal court against the State A motorist, seeking $100,000 in damages. The summons and complaint were served on a receptionist at the motorist’s place of business in State A. State A’s rules permit service of process in this manner, while State B’s rules do not.

If the motorist moves to dismiss the complaint on the basis of improper service of process, is the court likely to dismiss the action?

A

No, because the federal rules permit service under the rules of the state in which service will be effected.

  • The court is not likely to dismiss the action. Generally, Rule 4 allows for: (i) personal service, (ii) service left at the defendant’s usual place of abode with one of suitable age and discretion residing therein, or (iii) service upon an authorized agent of the defendant. Alternatively, service may be made as provided by the rules of the state in which the federal court sits or the state in which service is to be effected, regardless of the basis of subject matter jurisdiction. Here, the rules of State A, the state in which service was effected, permit service of process in this manner. Hence, service of process was proper.
How well did you know this?
1
Not at all
2
3
4
5
Perfectly
17
Q

Civ Procedure - Post Trial Matters

A car driver properly sued a truck driver for negligence in federal court for crashing into the driver’s car at an intersection. The truck driver had been issued a speeding ticket at the accident scene, after which he had mailed in the citation admitting he was speeding, paid the fine of $100, and never went to court.

At trial, the car driver wants to use the speeding citation as an admission of responsibility for speeding, so as to preclude the truck driver from relitigating whether he was speeding at the time of the accident.

Is the court likely to preclude the truck driver from relitigating the issue?

A

No, because he did not litigate the issue when he paid the citation.

  • The court likely will refuse to preclude the truck driver from relitigating the issue. For issue preclusion (collateral estoppel) to apply, the relevant issue must have been actually litigated and determined in the previous case. Here, although the truck driver admitted to speeding and paid the ticket, the issue was not actually litigated. Consent judgments and settlements generally do not carry with them preclusive effect. Therefore, issue preclusion will not apply.
How well did you know this?
1
Not at all
2
3
4
5
Perfectly
18
Q

Civ Pro - Post Trial Matters

A liquor store owner sued a woman for negligence in federal court after she drove her car through the front of his store. At trial, the store owner did not present any evidence on the issue of causation. No motions were filed during the trial, and the jury returned a general verdict for the plaintiff. Immediately after the verdict was read, the defendant filed a motion for judgment as a matter of law and a motion for a new trial.

The court may:

A; Deny the motion for a new trial, but grant the motion for judgment as a matter of law.
B: Deny both the motion for a new trial and the motion for judgment as a matter of law.
C: Grant either the motion for judgment as a matter of law or the motion for a new trial, but not both.
D: Grant the motion for a new trial, but not the motion for judgment as a matter of law.

A

D: Grant the motion for a new trial, but not the motion for judgment as a matter of law.

A motion for a new trial may be granted because of an error during the trial (usually going to the admissibility of evidence or the propriety of the jury instructions), because the verdict is against the weight of the evidence (limited to cases where the judge finds the verdict seriously erroneous), because of jury misconduct, or because the verdict is excessive or inadequate. A motion for judgment as a matter of law (formerly known as a motion for directed verdict) may be made by any party any time before submission of the case to the jury. Here, the court may grant the motion for a new trial under these circumstances because there was absolutely no evidence presented on the issue of causation, which is a fundamental element of a negligence claim that must be proven. As such, the court may find that the verdict is seriously erroneous and against the weight of the evidence and grant the motion for a new trial. However, the court cannot grant the motion for judgment as a matter of law because it was filed after the case was submitted to the jury, so it was too late for such a motion.

How well did you know this?
1
Not at all
2
3
4
5
Perfectly
19
Q

Con Law - Separation of Powers

One of the provisions of federal anti-smoking legislation imposes restrictions on federal economic development grants, which were awarded to states to promote and assist small businesses in urban areas. The legislation mandates that grants will be reduced by 10% for any state that fails to require businesses engaged in the sale of cigarettes to take steps to avoid sales to minors, including checking drivers’ licenses or photo ID cards. A tobacco-growing state that receives several million dollars under the federal grant program challenged the constitutionality of the provision in federal district court. The state established that the federal provision affects businesses that do not operate in interstate commerce.

Should the court uphold the federal provision?

A

Yes, because Congress may condition grants of money under its spending power.

  • The court should uphold the federal provision because it is within Congress’s power to spend for the general welfare. Article I, Section 8, provides that Congress may spend to “provide for the common defense and the general welfare.” This spending may be for any public purpose-not merely the accomplishment of other enumerated powers. Under this power, Congress may “regulate states by imposing explicit conditions on the grant of money to state or local governments.” Such conditions will not violate the Tenth Amendment merely because Congress lacked the power to directly regulate the activity that is the subject of the spending program as long as the conditions are (i) clearly stated, (ii) related to the purpose of the program, and (iii) not unduly coercive. Here, Congress has attempted to address a national problem-minors embarking on a potentially addictive habit that has been shown to damage health-by restricting access to the product causing the problem. Even if Congress’s enumerated powers would not permit it to directly require businesses to take the steps specified by the legislation, it may use its spending power to encourage states to impose these steps.
How well did you know this?
1
Not at all
2
3
4
5
Perfectly
20
Q

Con Law - 1st Amendment

To encourage minority business and foster pride in minority heritage, a state adopted legislation exempting magazines and other periodicals from the state’s receipts tax if 20% of the magazine is devoted to articles concerning minorities (a commission was set up to sample magazines to determine on a yearly basis whether they should be exempt). A publisher produced a sports magazine in the state that occasionally contained articles about minority athletes, but the commission determined that the publisher’s magazine was not eligible for the receipts tax exemption. After paying the tax assessed on her magazine, the publisher sued for a refund.

How will the court most likely rule?

A

In favor of the publisher, because the tax violates the First Amendment freedoms of speech and press.

  • The court should rule in favor of the publisher because the tax exemption regulates speech based on its content in violation of the First Amendment. The freedom of the press is guaranteed by the First Amendment. As with other areas within the First Amendment, the freedom does not prohibit all government regulation of the press, but it does place limits on regulation. The press and broadcasting companies can be subject to general business regulations and taxes, but generally may not be singled out for a special tax. Moreover, a tax impacting on the press or a subpart of the press cannot be based on the content of the publication absent a compelling justification. Although the state tax here appears to be a general receipts tax, the exemption is based on content, which means that the tax also is based on content (i.e., a publication is subject to the tax unless it contains . . . ). As discussed below, a compelling interest is not presented here, so the exemption is invalid and the tax should fail.
How well did you know this?
1
Not at all
2
3
4
5
Perfectly
21
Q

Con Law - Individual Rights

A state statute prohibited the state and any county, municipality, or other governmental unit within the boundaries of the state from hiring as a civil engineer any person who is not a citizen. A well-qualified engineer who is not a United States citizen read that the state’s department of transportation needed a new drafting engineer. The foreign engineer applied for the position and had the required qualifications. However, the hiring official turned down the engineer’s application, explaining that he could not hire her because of the state statute. The engineer filed suit in federal court, claiming that the statute violates her right to equal protection under the Fourteenth Amendment.

If the engineer prevails, what will be the most likely reason?

A

The state has failed to prove that the law is necessary to achieve a compelling government interest.

  • If the engineer prevails, it will be because the state has failed to show that the law is necessary to achieve a compelling state interest, as required by the Equal Protection Clause of the Fourteenth Amendment. Under that clause, a governmental action involving classification of persons will be subject to strict scrutiny if a suspect classification is involved. The law will be struck down unless the government proves that it is necessary to achieve a compelling interest. State and local laws that classify persons based on alienage are subject to strict scrutiny unless the law is discriminating against alien participation in the functioning of the state government. In that case, the law will be upheld as long as it is rationally related to a legitimate government interest. Thus, a state can validly refuse to hire aliens as primary or secondary school teachers or police officers because these positions have a direct effect on the functioning of government. On the other hand, a state law requiring citizenship for all civil service positions was held to be invalid. Similarly, a state law requiring a notary public to be a citizen was struck down under the strict scrutiny standard because a notary’s responsibilities are essentially clerical. The engineer could argue that the civil engineer position involves engineering skills rather than the functioning of government. If she prevails it will most likely be because the court agreed with her position and the state failed to meet its difficult burden under the strict scrutiny test of proving that the ban was necessary to achieve a compelling government interest. (
How well did you know this?
1
Not at all
2
3
4
5
Perfectly
22
Q

Con Law - Individual Rights

A philanthropist told his friend, who was a state governor, that he planned to build a museum. The governor thought that the museum would bolster the state’s tourism industry and offered to arrange to have the state purchase land and grant it to the museum to enable the philanthropist to build a bigger museum with his money than originally planned. The philanthropist agreed, and the museum was built.

The philanthropist undertook the hiring of the museum’s senior staff. He was of German descent and was ashamed of Germany’s actions during World War II. To assuage his own conscience, he refused to hire anyone whom he believed to be of German descent. A restoration expert applied for a job as chief curator of the museum, but the philanthropist refused to hire him because of his German background. The restoration expert discovered the philanthropist’s rationale and brings suit against the museum, claiming that the hiring practice violates his constitutional rights.

How is the court most likely to rule?

A

The policy is constitutional, because the museum is a private entity and so may constitutionally hire and fire as it desires.

  • The court should find that the museum is a private entity and that it may constitutionally hire and fire as it pleases because its actions do not constitute state action. The Equal Protection Clause prohibits states from discriminating against persons on the basis of race, alienage, or national origin unless the discrimination is necessary to achieve a compelling state interest. The museum’s policy here of not hiring persons of German descent clearly violates the Clause’s prohibitions. However, there is no constitutional violation here because there is no state action. The Equal Protection Clause prohibits only government infringement. This does not mean that only direct government action is proscribed. Private action may constitute state action where the private actor is performing an exclusive state function or the government is significantly involved in the private actor’s activities. The running of the museum here, however, is not an exclusive government function (e.g., running elections), and the state’s grant of the land for the museum does not constitute significant state involvement in the museum’s affairs (see below). Thus, there is no state action here and no constitutional violation. (Note that the museum’s actions probably violate several civil rights statutes that apply to private citizens.)
How well did you know this?
1
Not at all
2
3
4
5
Perfectly
23
Q

Con Law - Intergovernmental Immunities

A state statute prohibited the sale or possession of any food product containing more than one part per billion of a dangerous pesticide. An out-of-state driver taking her recreational vehicle through a corner of the state was stopped at a state inspection station. When the state trooper learned that the pantry of her RV was stocked with food, he asked to test a few samples of her baked goods. The samples contained about 600 parts per billion of the prohibited pesticide, and all of the other baked goods in her possession were tested and found to have the same level of pesticide. All of her baked goods, worth about $150, were confiscated and destroyed.

The state in which the driver lived has no laws governing the pesticide level of baked goods. A federal law designed to protect agricultural workers requires that any food product containing more than 500 parts per billion of the toxic pesticide must be labeled as such and be in special containers. The driver brings an action in federal court asserting that the state statute is invalid because it is preempted by the federal law.

How should the court rule as to this claim?

A

For the state, because the purposes of the federal law are different from those of the challenged statute.

  • The court should rule for the state because the purpose of the federal law is different from the purpose of the state law. The question here is whether the state law is preempted by the federal law. Implied preemption will be found where it was the intent of the federal government to occupy the entire field with its regulation, the state law directly conflicts with the federal law, or the state provisions prevent achievement of federal objectives. Because the federal law here is aimed only at occupational safety, no conclusion can be drawn that the federal government intended to occupy the entire field of regulation of pesticides, and the state law does not interfere with the federal law. For regulations involving health, safety, and welfare, the Court will presume that state police powers are not preempted unless that was the clear and manifest purpose of Congress when it enacted the federal law.
How well did you know this?
1
Not at all
2
3
4
5
Perfectly
24
Q

Con Law - 1st Amendment

A state legislature enacted a program by which students in the public schools could request instruction as to specific religions and religious beliefs, and thus participate in public school programs in which leaders of the religions involved gave religious instruction and performed religious practices on school grounds. The parent of a student who objected to religion being taught as part of the public school curriculum brought suit in federal court, seeking to have the program halted on First Amendment grounds.

Assuming the parent is a proper party to bring the suit, is the court likely to find the program violates the First Amendment?

A

Yes, because the program is not neutral toward religion and there is no long history of such religious instruction in public schools.

  • The strongest argument against the constitutionality of the program would be that the program is not neutral toward religion and that there is no long history of such religious instruction in public schools. It is the strongest argument because it reflects two of the criteria the Supreme Court uses in assessing whether government action violates the Establishment Clause: Is the government action neutral toward religion, and if not, is there a long history of the action being accepted or would the Founding Fathers have found the government action to be acceptable?
How well did you know this?
1
Not at all
2
3
4
5
Perfectly
25
Q

Con Law - PDP

Concerned about the rising death toll on the state’s highways, a state legislature enacted a statute providing for a summary one-year suspension of the driver’s license of any person convicted of three speeding violations within a 12-month period. The statute provided that an administrative hearing is immediately available upon request. However, that hearing is limited to a determination of whether the licensee is the same person who was convicted of the speeding violations.

A driver received three speeding citations in a three-week period and was convicted of all three charges. Her license was promptly suspended under the authority of the state statute. Without first seeking an administrative hearing, the driver files a suit in federal district court challenging the constitutionality of the statute.

Should the court uphold the constitutionality of the state law?

A

Yes, because the state’s interest in promptly removing unsafe drivers from its roads outweighs the driver’s right to a prior hearing under these circumstances.

  • The state law is valid because the prior judicial determinations that the driver violated the speeding laws satisfy the procedural due process requirements of the Fourteenth Amendment. Under the Due Process Clause of the Fourteenth Amendment, the state must provide some fair process or procedure before it may deprive a person of “life, liberty, or property.” Fair procedure at a minimum requires an opportunity to present objections to the proposed action to a fair, neutral decisionmaker. Whether a prior evidentiary hearing is required and the extent of procedural requirements is determined by weighing (i) the importance of the individual interest involved, (ii) the value of specific procedural safeguards to that interest, and (iii) the governmental interest in fiscal and administrative efficiency. [Mathews v. Eldridge (1976)] Because the government has taken control of who may drive automobiles on public roads, which is a sufficiently important area of human activity that persons have a liberty interest in it, the government must provide fair procedure to those who are specifically barred from engaging in the activity. In applying the Mathews v. Eldridge balancing test, the Court has held that the state generally must afford a prior hearing before a driver’s license is suspended or terminated. However, where the suspension is based on prior judicial determinations that traffic laws were violated, the driver has already had prior evidentiary hearings before unbiased decisionmakers on the significant factual issues involved. The governmental interest in keeping unsafe drivers off public roads and in not relitigating issues already fairly decided outweighs the driver’s interest in keeping her driver’s license. The procedural safeguards in the judicial proceedings in which she was convicted were sufficiently broad so that no additional prior hearing is necessary. Thus, the court should rule that the state law satisfies procedural due process requirements.
How well did you know this?
1
Not at all
2
3
4
5
Perfectly
26
Q

Con Law - 1st Amendment

Auto workers went on strike in a town heavily reliant on the auto industry. While negotiations between the union and management were ongoing, a person intercepted and recorded a phone call between the union’s president and management’s chief negotiator. A state statute makes it illegal to record a phone call without the consent of the parties being recorded. The statute also makes it illegal to play an illegally recorded conversation on television or radio.

The person who recorded the call anonymously sent the recording to a local TV station. The TV station news anchor played the recording on air.

Can the anchor who played the recording be prosecuted under the statute?

A

No, because the anchor did not record the conversation, and the information is truthful and about a matter of public significance.

  • The anchor may not be prosecuted. Generally, the press has a right to publish information about a matter of public concern, and this right can be restricted only by a sanction that is narrowly tailored to further a state interest of the highest order. The right applies even if the information has been unlawfully obtained in the first instance, as long as (1) the speech relates to a matter of public concern, (2) the publisher did not obtain it unlawfully or know who did, and (3) the original speaker’s privacy expectations are low. Here, because a strike is ongoing, the labor negotiation very likely is a matter of public concern, and the speakers could have anticipated great public interest in the conversation. Moreover, since each party to the conversation was speaking to an adversary, neither would have been justified in thinking the details of the conversation would be kept private. The anchor received the recording anonymously. Therefore, the anchor cannot be prosecuted.
How well did you know this?
1
Not at all
2
3
4
5
Perfectly
27
Q

Con Law - 1st Amendment

A state’s law requires driver’s licenses to display a photograph of the person whose name is on the license. The law was adopted to prevent fraudulent use of state drivers’ licenses as proof of identification. As such, the law does not provide for any exemptions from this requirement. Members of a religious sect within the state believe that allowing oneself to be photographed is sinful. A member of the sect who was refused a driver’s license because he would not allow himself to be photographed challenged the state regulation in federal court.

Is the court likely to uphold the application of the regulation to the religious group?

A

Yes, because it appears the law is religiously neutral, generally applicable, and not motivated by a desire to interfere with religion.

  • The law will be upheld because it is a neutral law that is applicable to all drivers in the state and nothing indicates it was motivated by a desire to interfere with religion. The Free Exercise Clause does not require exemptions from government regulations for a person whose religious beliefs prevent him from conforming his behavior to the requirements of the law. Unless the law was motivated by a desire to interfere with religion, it can be applied to regulate the conduct of one whose religious beliefs conflict with the law. Here, the sect member must allow his photograph to be taken if he wants to obtain a driver’s license; the state is entitled to enforce this regulation because it is a neutral law of general applicability.
How well did you know this?
1
Not at all
2
3
4
5
Perfectly
28
Q

Con Law - Separation of Powers

Congress enacted a statute that provided for direct money grants to the various states to be distributed by them to police agencies within their jurisdictions for the purpose of purchasing gas-efficient patrol vehicles. One of the objectives of the statute was to help reduce the dependency of the United States on imported oil.

Which of the following would provide the best constitutional justification for the statute?

A

The power to tax and spend for the general welfare.

(If general welfare and tax and spend are together, pick it. If general welfare if by itself, don’t pick it.)

  • The statute is authorized by Congress’s spending power. Article I, Section 8 gives Congress the power to spend “to provide for the common defense and general welfare.” This power allows Congress to spend for any public purpose as long as it does not infringe on other specific constitutional restrictions (such as the Bill of Rights). The statute here is clearly for a public purpose and is not otherwise unconstitutional; it is therefore within Congress’s spending power.
How well did you know this?
1
Not at all
2
3
4
5
Perfectly
29
Q

Con Law - Intergovernmental Immunities

One provision of a federal law provided that state governments may enact legislation regulating any form of pinball machine or video game, including location and hours of operation. In response, a Western state enacted legislation providing, among other things, that any video game sold or operated within the state use a particular LCD screen designed to minimize eyestrain.

A corporation that designs and manufactures video games for sale throughout the United States and in Europe is based in the Midwest. Approximately 10% of its gross sales are made in the Western state that has regulated the LCD screens. The corporation’s machines are not manufactured using the special eye-protecting LCD screens; to install such screens in all machines manufactured would cause the price of the machines to increase by 20%, and to use the screens in machines sold only in the Western state would increase the cost of those machines by 50%. The corporation files suit in federal court to enjoin enforcement of the state video game statute.

How should the court rule?

A

For the state, because Congress has acted within its power to authorize video game regulation by the states.

  • The state will prevail because Congress has consented to the state regulation of interstate commerce. A state may regulate local aspects of commerce (i.e., intrastate commerce), but state regulation that discriminates against or substantially burdens interstate commerce may be held invalid under the Supremacy Clause, because of Congress’s very broad power to regulate interstate commerce under the Commerce Clause. Here, the state statute, standing alone, might have been held invalid because its substantial burden on interstate commerce could have been found to outweigh any legitimate local interest in reducing eyestrain. However, the federal statute changes the equation. Because Congress’s power over interstate commerce is very broad, Congress may allow a state to adopt legislation that would otherwise be invalid as an unconstitutional burden on interstate commerce; this is what Congress did here. By allowing the state regulation, it is actually exercising the federal commerce power-it simply allows for nonuniform (state-by-state) rules.
How well did you know this?
1
Not at all
2
3
4
5
Perfectly
30
Q

Con Law - Judicial Review

Recently enacted legislation required farmers in certain counties of a western state to use drip irrigation systems instead of traditional methods in order to conserve water for agricultural and other uses. A farmer who refused to use the drip system was charged pursuant to the enforcement provisions of the legislation. A state court enjoined him from using other irrigation methods and fined him.

The farmer appealed to the state supreme court, renewing his trial court claims that the irrigation legislation violated a state constitutional provision prohibiting certain governmental intrusions into private commercial activities and that it was preempted by federal water management statutes. The state supreme court held that the state constitution prohibited the challenged legislation, and construed the relevant statutes as being within the parameters of the federal statutes, and thus preempted.

If the state petitions for certiorari to the United States Supreme Court, how should the Court rule on the petition?

A

Deny the petition, because there is no substantial federal question that is dispositive of the case.

  • The state’s challenge to the decision of the state supreme court presents no substantial federal question and will be denied. The Supreme Court’s appellate jurisdiction under 28 U.S.C. section 1257 extends to reviewing the decision of the highest court of a state where the validity of state legislation is called into question on the ground that it is unconstitutional or contrary to federal statutes. However, the Supreme Court will hear a case from a state court only if the state court judgment turned on federal grounds. The Court will refuse jurisdiction if it finds adequate and independent nonfederal grounds to support the state decision, because a different interpretation of the federal statutes would have no effect on the judgment rendered by the state court, so that the Supreme Court, in effect, would be rendering an advisory opinion. Here, even if the state court was incorrect in holding that federal statutes preempted the state legislation, it also held that the state constitution prohibited the state legislation. Hence, a different interpretation of the federal preemption issue would have no effect on the outcome of the case.
How well did you know this?
1
Not at all
2
3
4
5
Perfectly
31
Q

Con Law - Judicial Review

A state prohibited the sale of heated grips for motorcycles within its borders to discourage motorcyclists from riding in dangerously cold conditions. The owner of a cycle shop that sells heated grips within the state that are purchased from an out-of-state manufacturer seeks an injunction in state court prohibiting the state from enforcing its statute. The owner claims that the statute unreasonably interferes with interstate commerce.

If the state court rules that the relevant statute is valid and denies injunctive relief, which of the following is the proper next step for the owner to take to obtain review of the state court decision?

A

Appeal to the state appellate courts.

  • The proper next step for the corporation is to appeal to the state appellate courts. A plaintiff bringing an action in a state trial court is required to exhaust its state appellate remedies before seeking review in federal courts, even where federal issues are involved.
How well did you know this?
1
Not at all
2
3
4
5
Perfectly
32
Q

Con Law - Judicial Review

Federal legislation provided that the marketing and sale of oranges was subject to the control of a local marketing authority. The marketing authority determined what quantity of oranges could be sold by each grower, the price, and the location of sale. These decisions were made by a council of local growers whose members were selected by the federal Department of Agriculture. The applicable federal legislation provided, in part, that when any grower subject to a marketing order challenged the propriety of that order, the council of the marketing authority must submit the controversy to the United States district court with geographical jurisdiction for a recommendation as to whether the order should be confirmed, modified, or rescinded. After the hearing in district court, the council must revote on the challenged marketing order.

A citrus grower brings suit in United States district court, seeking on constitutional grounds to enjoin enforcement of the federal legislation providing for the marketing order that the council issued with regard to his orange crop.

If the court rules on the constitutional issue, is the grower likely to prevail?

A

Yes, because the federal legislation permits the federal district court to give an advisory opinion.

  • Because the federal legislation merely allows the district court to issue a “recommendation,” the legislation permits the rendition of advisory opinions. Article III of the United States Constitution establishes the basis for the judicial power of federal courts. It provides that the judicial power extends to “cases and controversies.” Although Congress has power to delineate the jurisdictional limits of Article III courts, it is bound by the standards of judicial power set forth in Article III as to subject matter, parties, and the requirement of “case or controversy.” Thus, Congress cannot require these courts to render advisory opinions or perform administrative or nonjudicial functions. The federal orange marketing legislation at issue here does not give federal district courts the authority to render binding decisions in final resolution of a controversy. Rather, the legislation simply allows the courts to make a recommendation as to confirmation, modification, or rescission of a challenged marketing order. This recommendation is apparently nonbinding on the parties and is followed by a re-vote of the marketing council. These circumstances indicate that, under this federal legislation, an Article III federal court would be rendering an advisory opinion in violation of the Constitution.
How well did you know this?
1
Not at all
2
3
4
5
Perfectly
33
Q

Con Law - Judicial Review

A merchant owned a skate rental business that she operated out of a specially equipped van. She would drive to various parks and public beaches within her home state and rent roller skates, related safety equipment and lightweight stereo/earphone sets to passersby on an hourly basis. She also sold skates and skating equipment. About 50% of the merchant’s time is spent in a single city, and she earns about 70% of her gross rental and sale income at that city’s beach areas. After receiving numerous complaints from beachgoers about the sidewalks congested with roller skaters, the city council passed an ordinance prohibiting roller skating on public property between the hours of 7 a.m. and 9 p.m.

If the merchant seeks to enjoin enforcement of the ordinance in federal district court on the basis that it is unconstitutional, what should the court do?

A

Reach the merits of the merchant’s challenge, because enforcement of the ordinance will harm her business and the rights of the public are linked to her rights.

  • The merchant has standing to challenge the ordinance because her business will be harmed by it, and this will adversely affect her relationship with those who would buy and rent her equipment, resulting in an indirect violation of their rights. A person who challenges a government action must have standing to raise the constitutional issue. A person has standing only if she can demonstrate a concrete stake in the outcome of a controversy and that the governmental action at issue impairs her own rights. A plaintiff may assert third-party rights if she has suffered injury and that injury adversely affects her relationship with third parties, resulting in an indirect violation of their rights. Enforcement of the ordinance at issue will effectively destroy 70% of the merchant’s business. Thus, the merchant is faced with an immediate and direct threat of injury to her livelihood as a result of the ordinance. This injury will also adversely affect her relationship with those persons at the beach areas of the city who would normally buy and rent her equipment, because they will now be prohibited from roller skating on the beaches between the hours of 7 a.m. and 9 p.m. This will, in turn, cause a potential violation of the rights of such third parties (e.g., the law might violate their First Amendment right to join with other persons for expressive activity). Consequently, the merchant is deemed to have standing to challenge the ordinance, and the court will reach the merits of the challenge.
How well did you know this?
1
Not at all
2
3
4
5
Perfectly
34
Q

Con Law - Individual Rights

A state’s pension program provided supplemental state pension benefits to surviving spouses and children of state employees. The program provided that when the spouse remarried, that spouse’s benefits would be gradually terminated based on a statutory formula. Because of statistics showing past disparities between the household income levels of male surviving spouses and female surviving spouses, different formulas were used for the termination schedule depending on whether the surviving spouse was male or female.

A widower of a state employee was informed after he remarried that his pension benefits would be terminated in 90 days according to the applicable formula. Upon learning that a similarly situated widow would have continued to receive benefits for six months after remarrying, the widower decided to file suit in federal court, alleging that the state program is unconstitutional because it is discriminatory and it unfairly burdens his right to marry.

Which of the following best states the burden of persuasion in this case?

A: The state must demonstrate that the program is narrowly tailored to achieve a compelling government interest.
B: The state must demonstrate that the program is substantially related to an important government interest.
C: The widower must demonstrate that the program is not substantially related to an important government interest.
D: The widower must demonstrate that the program is not rationally related to a legitimate government interest.

A

B: The state must demonstrate that the program is substantially related to an important government interest.

  • The state has the burden of proving that the program is substantially related to an important government interest. When analyzing government action based on gender, the courts will apply an intermediate standard of review and strike the legislation unless it is substantially related to an important government interest. In these cases, the government bears the burden of proving this substantial relationship. Here, because the formula used to calculate termination of pension benefits depended on whether the surviving spouse was male or female, the legislation discriminates on the basis of gender. Thus, an intermediate scrutiny standard will be applied.
How well did you know this?
1
Not at all
2
3
4
5
Perfectly
35
Q

Con Law - Intergovernmental Immunities

Congress passed a statute designed to make college tuition-free. The statute significantly increased funding to states for this purpose; however, it made all federal funding to states dependent on the state making state universities tuition-free to United States citizens, and several state budgets consist of as much as 50% federal funding. Some states do not wish to participate in the program and want to retain their level of federal funding that existed before the statute was passed.

If these states challenge the constitutionality of the statute in court, will the court likely uphold the statute?

A

No, because the funding condition is unduly coercive.

  • The court will not uphold the statute. Congress may regulate states through the spending power by imposing conditions on the grant of money to state governments. These conditions do not violate the Tenth Amendment merely because Congress lacked the power to directly regulate the activity so long as the conditions are clearly stated, relate to the purpose of the program, and are not unduly coercive. Here, the condition is unduly coercive because the funding amounts to as much as 50% of states’ total budgets.
How well did you know this?
1
Not at all
2
3
4
5
Perfectly
36
Q

Con Law - Individual Rights

The National Park Service recently created a new personnel level for field employees, which became the highest salaried position available to Park Service field employees. The position is restricted to employees over six feet in height. A female ranger who is five feet, three inches tall seeks your advice as to whether she can challenge the validity of the height restriction in federal court.

If you decide to file suit on her behalf, which of the following would be your strongest argument against the validity of the restriction?

A

Because most women are less than six feet tall, the restriction is an invalid discrimination on the basis of gender in violation of the Due Process Clause of the Fifth Amendment.

  • The ranger’s strongest argument, although by no means guaranteed of success, is that the height restriction is a gender-based classification that is not substantially related to important governmental interests. The Due Process Clause of the Fifth Amendment protects against action by the federal government. Although not expressly stated, this clause also provides an equal protection guarantee against federal action that generally applies to a similar extent that the Fourteenth Amendment Equal Protection Clause applies to the states. If the ranger can show that the Park Service restriction actually establishes a classification of eligibility for the new position based on gender, then the restriction will be found to violate the Fifth Amendment Due Process Clause unless the government has an exceedingly persuasive justification that the restriction is substantially related to important governmental interests. However, if the ranger is only able to show that the restriction has a discriminatory impact without being able to prove discriminatory intent, the court will not treat it as a gender-based classification and the ranger will not be successful. Despite the difficulty of success, however, (B) is the correct answer because it provides the best possibility of a winning argument for the ranger. Remember that “best of the lot” questions are sometimes “best of a bad lot.” You might need to work through all of the alternatives and arrive at the best answer by process of elimination.
How well did you know this?
1
Not at all
2
3
4
5
Perfectly
37
Q

Con Law - Intergovernmental Immunities

A state study shows that computer chip manufacturing has decreased 60% in the past five years due to a large supply of foreign chips entering the state. To prevent the complete loss of computer chip manufacturing, the state enacted legislation requiring that at least 50% of the units sold by electronic products retailers within the state incorporate chips manufactured within the state. The owner of a chain of computer stores in the state sells electronic devices manufactured entirely in other states.

If the computer store owner challenges the state legislation on constitutional grounds, will she likely prevail?

A

Yes, because the legislation discriminates against out-of-state commercial activity.

  • The store owner likely will prevail because the state legislation discriminates against out-of-state goods in violation of the negative implications of the Commerce Clause. A state may regulate local aspects of interstate commerce if such regulation is not in conflict with federal regulations and if: (i) the subject matter of the regulation does not require nationally uniform regulation; (ii) the regulation does not discriminate against out-of-state competition to benefit local economic interests; and (iii) any incidental burden on interstate commerce of the nondiscriminatory regulation does not outweigh the legitimate local benefits produced by the regulation. Laws that are designed to protect local businesses against interstate competition generally will be invalidated. Here, the state legislation discriminates against out-of-state manufacturers to protect local businesses by limiting the amount of units that can be sold by retailers of electronic products containing chips manufactured out of state. There is no such limit on units that can be sold that utilize locally manufactured chips. This law was enacted to encourage the development of local manufacturing operations. Because this law is designed to protect local businesses against interstate competition, it should fail a challenge under the Commerce Clause.
How well did you know this?
1
Not at all
2
3
4
5
Perfectly
38
Q

Contracts - Consideration

A jogger found a stray dog in the park. She took the dog home with her and placed an ad in the paper to try to find the dog’s owner. Soon thereafter, the owner of the dog contacted the jogger. He came to the jogger’s home and identified the dog as his. He offered to pay the jogger a $200 reward at the end of the week. The jogger thanked the dog owner but turned down the reward.

At the end of the week, however, the jogger changed her mind, so she called the dog owner and told him that she would like the reward after all. He refused to pay her, and she sues him for breach of contract.

What will the jogger recover?

A

Nothing, because there was no consideration to support a contract.

  • The jogger will recover nothing because her finding the lost dog occurred prior to the dog owner’s promise to pay the $200. An enforceable contract must be supported by consideration. Consideration consists of: (i) a bargained-for exchange between the parties; and (ii) an element of legal value to that which is bargained for. Legal value is present if the promisee has incurred a detriment (i.e., has done something she is under no legal obligation to do or has refrained from doing something that she has a legal right to do). To have a “bargained-for exchange,” the promise must induce the detriment, and the detriment must induce the promise. If something has already been given or performed before the promise is made, it will not satisfy the bargain requirement, because it was not given in exchange for the promise. Here, the jogger was under no legal obligation to return the dog to its owner. Thus, in doing so, she incurred a detriment. However, the jogger was not induced to so act by the dog owner’s promise to pay $200. Because the jogger’s actions regarding the dog were performed before the dog owner’s promise, those actions were not given in exchange for the promise when made. Thus, the “bargain” element is absent.
How well did you know this?
1
Not at all
2
3
4
5
Perfectly
39
Q

Contracts - 3rd Party Beneficiary

A large insurance company instituted a supplemental benefit plan for its own employees. Under the plan, any employee who had worked for the company for at least 25 years would be permitted to designate a charity to receive, on the employee’s retirement, a donation in the employee’s name of six months’ worth of the employee’s salary. The plan gave participating employees an unqualified right to change the beneficiary at any time before payment was made. An employee nearing retirement enrolled in the plan and named his favorite church as the beneficiary of the donation. The church received a letter from the company informing it that the employee had named it beneficiary of his plan and indicating the approximate amount that it would receive upon the employee’s retirement in 10 months. The letter did not inform the church of the employee’s right to change beneficiaries before that time. Church elders, anticipating the gift, authorized restoration work to the church building, making plans to pay for the work with the funds from the employee’s benefit program.

Six months later, the employee converted to a different religion and changed the beneficiary of his plan to his new church. When the employee retired, the company paid the benefit to his new church. His old church, which had paid for the restoration work on its completion, demanded payment of the benefit from the company. When payment was refused, the church sued the company.

Which party is likely to prevail?

A

The church, because the interests of justice require it.

  • The church will be able to recover against the insurance company because the interests of justice require it. Under the majority view, consideration is not necessary to make an agreement at least partially enforceable where the facts indicate that the promisor should be estopped from not performing. This is stating the concept of promissory estoppel without labeling it as such. Under the Second Restatement, a promise is enforceable to the extent necessary to prevent injustice if the promisor should reasonably expect the promise to induce action or forbearance and such action or forbearance is in fact induced. Here, the insurance company sent a letter to the church informing it that the employee had named the church beneficiary under his employee benefits program. The company did not warn the church that the employee had the right to change his beneficiary and should have reasonably expected that the church would rely on the promise in some way; it is not necessary in charitable contribution cases that the promisor know of a specific expenditure that the recipient made or is going to make. The church did in fact rely on the promise by authorizing and paying for the restoration work. Hence, to prevent injustice, as choice (A) states, the church can recover against the company to the extent of its reliance (the cost of the restoration work).
How well did you know this?
1
Not at all
2
3
4
5
Perfectly
40
Q

Contracts - Consideration

Two brothers who were certified public accountants worked together at a large accounting firm practicing their chosen profession. The older brother was concerned about his younger brother’s apparent inability to show up at his job by 9 a.m. each morning, sober and clear-eyed. One day, after the younger brother showed up late for work yet again, the older brother told him that if he would show up at the office sober and ready to work by 9 a.m. each morning for the next 10 months, he would pay him $15,000 at the end of that time. The younger brother accepted the offer and complied with its terms from that day forward. Nine months later, the older brother died unexpectedly. One month after that, the younger brother filed a claim with his brother’s estate for the $15,000.

Will the younger brother prevail in his claim?

A

Yes, because he has performed under a valid contract, and thus his brother’s estate must now perform.

  • The younger brother will prevail because he has performed under a valid contract. He entered into and performed a valid unilateral contract with his brother, who offered to give him $15,000 if he showed up at the office sober and ready to work by 9 o’clock each morning for the next 10 months. He accepted by fully performing; his giving up the right to do something that he had a legal right to do constitutes valid consideration. Because the younger brother fully performed his duties under the contract, the older brother’s estate is bound to perform his duties and must now pay him.
How well did you know this?
1
Not at all
2
3
4
5
Perfectly
41
Q

Contracts - Defenses

A nephew asked his uncle, who like him was a farmer, to guarantee a loan to buy a new tractor. The local bank had already refused to extend credit to the nephew alone to buy the tractor. The uncle was inclined to refuse, but then decided that he could benefit from his own use of the tractor, so he told his nephew that he would guarantee the loan if he could use the new tractor without cost for 10 days during his harvest season. The nephew agreed to his uncle’s proposal. The uncle went to the bank and told the loan officer that he was willing to guarantee the proposed loan to his nephew. This prompted the loan officer to agree to extend the requested credit to the nephew. Although the loan officer did not make the uncle sign any papers, the uncle provided consideration and the bank issued the nephew a loan commitment statement. That evening, the uncle had a change of heart. The next day, he telephoned the loan officer and told him to forget about his guaranteeing any loan to his nephew. Despite the uncle’s phone call, the loan officer did not stop the check from being issued, and the nephew received the money to purchase the tractor. He drove the tractor over to the uncle’s farm and delivered it for the uncle’s 10-day use, as promised. The uncle told his nephew that he did not want to use the tractor and that he was not guaranteeing his loan. Within six months, it became clear that the nephew could not make good on the loan.

If the bank sues the uncle for the unpaid portion of the loan, who will likely win?

A

The bank, because the uncle’s main purpose in making the agreement with the bank was to benefit himself, not his nephew.

The uncle’s main purpose in making the agreement was to benefit himself rather than his nephew, so the agreement is outside of the Statute of Frauds and is enforceable even though it was oral. Under the Statute of Frauds, certain agreements must be evidenced by a writing that contains: (i) the identity of the party sought to be charged; (ii) identification of the contract’s subject matter; (iii) terms and conditions of the agreement; (iv) recital of consideration; and (v) signature of the party to be charged, or of his agent. One type of agreement that is covered by the Statute of Frauds is a promise to answer for the debt or default of another where the promise is collateral rather than primary. However, where the main purpose or leading object of the promisor is to secure an advantage or pecuniary benefit for himself, the contract is not within the Statute of Frauds, even if the effect is still to pay the debt of another. The uncle guaranteed the loan to his nephew, which means that the uncle agreed to repay the loan only if his nephew refused to do so. Therefore, the uncle made a collateral promise to answer for the debt or default of the nephew regarding the loan from the bank. However, because the main purpose of the uncle’s making the agreement was to benefit himself rather than his nephew (by, e.g., being allowed to use the tractor without cost), the agreement is outside the scope of the Statute of Frauds and would be enforceable against the uncle even in the absence of a writing.

How well did you know this?
1
Not at all
2
3
4
5
Perfectly
42
Q

Contracts - Defense (Parol Evidence)

A retail store owner e-mailed an electronics supplier an offer to buy 300 flash drives imprinted with the retailer’s trademark at $2 per flash drive. The supplier immediately e-mailed back his acceptance. A week later, the retailer telephoned the supplier and truthfully explained that she had made an error in her order-it should have been for 200 flash drives. She asked if she could reduce her order to 200. The supplier thought the imprinting process was not yet completed and so agreed to reduce the order to 200. After the phone call, the supplier discovered that all 300 flash drives had already been imprinted. He delivered the 300, but the buyer accepted only 200.

Can the supplier recover damages with respect to the 100 flash drives that were not accepted?

A

No, because the supplier agreed to the modification and his mistake was unilateral.

  • The supplier cannot recover damages for the flash drives that were not accepted because the modification is valid and enforceable. For sale of goods contracts, a modification must be in writing if the contract as modified falls within the Statute of Frauds; i.e., if the contract as modified is for $500 or more. Here, the buyer and the supplier’s original contract was for $600 worth of flash drives, but the modified contract is for $400 worth of flash drives. Because the contract as modified is for less than $500, no writing is required to make the modification enforceable. Moreover, a unilateral mistake is not a defense unless the other party knew or should have known of the mistake. Here, the supplier was mistaken as to whether the imprinting was done, but the mistake was unilateral-the buyer did not know that the imprinting was done and had no reason to know. Thus, the supplier’s mistake was unilateral and not a defense. Therefore, the contract is enforceable only to the extent of the 200 flash drives accepted.
How well did you know this?
1
Not at all
2
3
4
5
Perfectly
43
Q

Contracts - Consideration

A debtor owed a creditor $1,200 on a promissory note that was due on August 1. After the debtor told the creditor that he might not be able to pay the note on its due date, the creditor agreed to extinguish the debt if the debtor, who was the manager of a discount electronics store, bought a new entertainment system that sold for $1,200 and had it delivered to the creditor’s home by August 15. Because the debtor would have to pay only $600 for the system due to his manager’s discount, he agreed and the parties signed a written contract on July 26.

Is the new agreement between the debtor and the creditor legally enforceable?

A

Yes, because the debtor incurred a different obligation than he originally had.

  • The new agreement between the two parties is enforceable as an accord. An accord is an agreement in which one party to an existing contract agrees to accept, in lieu of the performance that he is supposed to receive from the other party, some other, different performance. Generally, an accord must be supported by consideration, but the consideration may be of a lesser value than the originally bargained-for consideration in the prior contract, as long as it is of a different type or the claim is to be paid to a third party. Here, the debtor’s obligation to provide the creditor with a new entertainment system was a sufficient new consideration to form a valid accord.
44
Q

Contracts - Conditions, Discharge, Breach

A buyer of a new car owed the car dealership where she purchased the vehicle $1,000 on a promissory note that was due on December 30. The buyer determined that she would be unable to pay the note on its due date, and she informed the owner of the dealership of that fact. The owner told her that she would not have to pay the debt if she bought him four tickets to a popular concert on January 15 that had been sold out for weeks, because she worked as publicist for the concert venue. She agreed to do so, and the parties memorialized their agreement in a signed writing on December 18. On January 2, the dealership filed suit against the car buyer for failure to pay the $1,000 promissory note, before the car buyer had secured the concert tickets for the owner.

May the car buyer have this action enjoined by introducing evidence of the December 18 agreement?

A

Yes, because the December 18 agreement between the parties suspended the car buyer’s obligation on the promissory note.

  • The car buyer may enjoin the dealership’s action because the dealership currently does not have the right to enforce the promissory note. A valid accord, taken alone, does not discharge the prior contract. It merely suspends the right to enforce it in accordance with the terms of the accord contract. The performance of the accord agreement, which is called satisfaction, discharges not only the accord agreement but the original contract as well. Where the accord agreement is breached by the creditor by suing on the original contract, as is the case here, the debtor may seek to have the action enjoined by raising the accord agreement as an equitable defense.
45
Q

Contracts - 3rd Party Beneficiaries

A large-scale bakery in the South entered into a written contract with a commercial apple orchard in the upper Midwest to purchase 200 bushels of apples at a cost of $8 per bushel. The contract provided that the apple orchard would deliver the apples “F.O.B. Louisville Railroad Depot,” where the apples would be loaded onto a train headed south. The orchard assigned all of its rights under the contract to a large produce distributor which, in turn, hired a trucking company to deliver the apples to Louisville. En route to Louisville, the truck skidded off the road due to inclement weather and overturned, and the apples were destroyed. The bakery brought suit against the apple orchard for breach of contract.

What will be the probable outcome of the litigation?

A

The bakery will recover the amount necessary to replace the destroyed apples, over the contract price.

  • If the bakery brings an action against the apple orchard, the bakery will be able to recover the costs of replacing the destroyed apples because the apple orchard remained liable on the assigned contract and it had the risk of loss. Although most contractual duties may be assigned-unless they are personal-and the obligee must accept performance from the delegate, the delegating party (delegator) remains liable on his obligation. Thus, an assignment of a contract that includes a delegation of duties does not relieve the assignor from its duty to perform. Here, the bakery did not receive the performance that was due (the apples), so it could sue the apple orchard to recover for the breach. When a nonbreaching buyer does not receive the contracted goods, it has several options: it can cancel the contract and recover any incidental damages, or it can purchase replacement goods and sue for the cost of replacement-“cover.” Damages under the latter option are measured by the difference between the contract price and the amount the buyer actually has to pay for the replacement goods.
46
Q

Contracts; Conditions; Breach

An applesauce bottler wishing to redesign his factory entered into a written contract with a contractor. The contract provided that the contractor would design and install a new glass bottle system, replacing the plastic bottle system, by March 10, and the bottler would pay the contractor a total of $100,000. The contract provided for a first payment of $50,000 on completion of the design plans and a second payment of the balance after installation and successful testing of the system. The contractor presented the bottler with the finished design plan in January, and the bottler paid him $50,000. The contractor ran into difficulty procuring the parts called for by his design, and this delayed the installation. The installation and testing were completed on March 25.

Disappointed by the delay, the bottler now refuses to pay the contractor anything further. If the contractor sues the bottler, which party is likely to prevail?

A

The contractor, because the contract did not provide that time was of the essence.

  • The late installation does not justify the refusal to pay. Unless the nature of the contract is such as to make performance on the exact day agreed upon of vital importance (e.g., contract for use of a wedding chapel), or the contract by its terms provides that time is of the essence, failure by a promisor to perform at the stated time will not be material. Merely providing a date for performance does not make time of the essence. Nothing in the contract here states that time is of the essence, and the contract does not by its nature require timely performance. This is a services contract, rather than a goods contract governed by the UCC, so perfect performance is not required. Thus, late performance is treated as a minor breach that gives the nonbreaching party a right to damages but does not relieve him of his duty to perform. The bottler still has the duty to pay the $50,000 but would be entitled to an offset for the damages suffered due to the delay.
47
Q

Contracts - Offer & Acceptance

On March 1, the purchasing agent for a suburban school district sent a “quotation request form” to a supplier of school furniture requesting an offer for the sale of 20 student chairs. The form was on school district letterhead and signed by the purchasing agent. It specified that the offer must be held open for four months and that the price term must be no higher than $30 per chair. The supplier telephoned the purchasing agent and told him that he would sell the school district 20 chairs at $20 per chair. He also agreed to hold the offer open for four months. The purchasing agent thanked the supplier for the offer and indicated that he would get back to him within that time period. On May 1, before the purchasing agent had responded to the supplier’s offer or taken any action in reliance on it, the supplier e-mailed the purchasing agent stating that demand for student chairs had been higher than expected and that the offer was terminated. On May 2, the purchasing agent called the supplier, told him that the school district was treating his offer as still being open, and accepted it on its terms.

Did the purchasing agent’s call on May 2 create a legally enforceable contract with the supplier?

A

No, because the supplier did not sign the form specifying the length of time that the offer would be held open.

  • No contract was created because the supplier effectively revoked his offer. Under the UCC, an offer by a merchant to buy or sell goods in a signed writing that, by its terms, gives assurances that it will be held open is not revocable for lack of consideration during the time stated (not to exceed three months). If the term assuring that the offer will be held open is on a form supplied by the offeree, it must be separately signed by the offeror. Here, the school district supplied the form stating that the offer must be held open for four months. The supplier’s verbal assent to that requirement was not sufficient to qualify as a firm offer under the UCC. Thus, he was free to revoke his offer.
48
Q

Contracts - Remedies

A builder contracted with a landowner to construct a house on the landowner’s property for a contract price of $100,000. Before the builder completed his performance, the landowner informed the builder that he (the landowner) no longer wanted the house, and that he had no intention of paying the builder. At that time, the builder had already incurred costs of part performance of $30,000 and would have to spend an additional $60,000 to finish the job.

If the builder sues the landowner for breach of contract, what is the builder’s most likely recovery?

A

$40,000.

  • The builder is most likely to recover $40,000 because he is entitled to his profit plus costs. Where an owner breaches a construction contract after construction has been started but before construction is completed, the builder is entitled to recover any profit he would have derived from the contract plus any costs he has incurred to the date of the breach. Here, the builder’s profit would have been $10,000 and his costs up to the time of the breach are $30,000. Thus, he can recover $40,000. Another way of saying this is contract price minus the cost of completion. Here, the contract price was $100,000 and the cost of completion was $60,000. Thus, the builder is entitled to $40,000.
49
Q

Contracts - Defenses

A 17-year-old boy walked into a medical clinic and requested assistance with a deep cut he received when he fell off his bike. The doctor told the boy that if he agreed to work at the clinic for 45 hours a week for four weeks, he would stitch the wound and apply a medicated bandage. The clinic typically charges $225 for such treatment. Although it seemed like a lot of work for $225, the boy needed immediate medical attention, so he accepted the offer and promised to report for work the next day, after which the doctor treated the boy’s injury. On his way home from the clinic, the medicated bandage fell into a ditch and was lost for good. The boy refused to report for work the next day.

The medical clinic sues the boy to recover the costs of the medical treatment.

If the court rules in favor of the boy, what is the likely reason?

A

The contract was unconscionable.

  • If the court rules in favor of the boy, it will be because the parties’ contract was unconscionable. Unconscionability usually arises where one of the parties has substantially superior bargaining power and can dictate the terms of the contract to the other party, who has inferior bargaining power. Here, the boy told the doctor that he needed medical attention and the doctor forced him to agree to work for, in essence, $1.25 per hour in exchange for care and a medicated bandage. These facts would meet the requirements of an unconscionable contract.
50
Q

Contracts - Conditions; Discharge; Breach

A chef wanted to open his own restaurant and a contractor offered to build the place for $160,000. Their written contract provided that the chef would pay the contractor $60,000 in cash when construction commenced, scheduled for April 15 after the spring thaw. On completion of the restaurant on September 30, the contractor would be paid the remaining $100,000. The region had a late spring, and on April 30 the contractor had not yet commenced construction of the restaurant.

Has the contractor breached the contract?

A

No, and the chef need not make the initial $60,000 payment.

  • The contractor has not breached the contract, and the chef need not make the first payment until the contractor begins work. The contractor promised to build the restaurant by September 30. He did not promise to begin on April 15. The contractual term as to the contractor’s beginning construction is a condition precedent to making the first payment. It is a condition relating to the chef’s performance, not the contractor’s performance. Nothing in the facts indicates that the contractor promised to begin on April 15. Contracts are construed as a whole and words are given their ordinary meaning. The purpose of the contract is to build a restaurant by September 30. Construction cannot commence before the spring thaw. Thus, the best interpretation is that the language regarding commencement of construction was merely a condition of the chef’s first payment, inserted to insure that the contractor was motivated to begin and that the chef would not be out of pocket if the contractor failed to begin. Thus, the term regarding the beginning of construction of the restaurant on April 15 merely fixes a tentative time of the start of performance, and does not involve an absolute promise by the contractor to commence performance on April 15. Because the contractor was under no absolute duty to commence construction on April 15, his failure to do so does not constitute a breach of the contract.
51
Q

Crim Pro - Arrest, Search & Seizure

Federal narcotics officers suspected the defendant of growing marijuana in his greenhouse, which was connected to his house. The narcotics officers learned from an anonymous informant that the semi-opaque panes of glass on the greenhouse were being replaced during the night with a newer type of glass that let in more light without an increase in visibility. Without a warrant, the officers flew over the defendant’s greenhouse in a helicopter that night. One of the officers focused on the greenhouse with a pair of “night vision” thermal imaging binoculars supplied by the Department of Defense and not available to the general public. He determined that marijuana was being grown. The officers then went to a magistrate, swore out a warrant, and arrested the defendant.

If the defendant moves to suppress any evidence gathered by virtue of the flyover, how should the court rule on the motion?

A

Grant it, because the “night-vision” binoculars were not available to the general public.

The use of thermal imaging binoculars to observe the marijuana where it could not be observed by simply using the naked eye likely renders the search invalid. To be able to assert a Fourth Amendment right, a person must have a reasonable expectation of privacy with respect to the place searched or the item seized. There is no such expectation of privacy in objects or places held out to the public or that may be viewed from a public vantage point. Thus, the police may fly over an area to observe it with the naked eye, and even a low flyover by a helicopter to view inside a partially covered building is permissible. This is true even if the area is within the curtilage. However, the police may not use technologically enhanced methods that are not available to the public to search areas (at least as to areas within the curtilage). In the instant case, the police have flown over the defendant’s greenhouse at night and used a means of enhancing their vision that is not available to the general public. This enabled them to see what could not have been observed with the naked eye, which likely constitutes an impermissible search. [See Kyllo v. United States (2001)]

52
Q

Crim Pro - Right to Counsel; Confessions

A police officer went to the defendant’s house and placed him under arrest for operating an auto theft ring. As the defendant was being arrested, he told his wife, “You had better call our lawyer; I don’t want to sign anything unless she’s with me.” The defendant was given Miranda warnings on the way to the police station. Meanwhile, the defendant’s lawyer called the station and told the desk sergeant that she was on her way and to have the defendant call her as soon as he arrived. The sergeant assured her that the defendant would be held without questioning for several hours until the district attorney arrived. When the defendant arrived at the station, the arresting officer and another officer immediately put the defendant in an interrogation room and questioned him about a bank robbery that had taken place two days ago. They did not inform him of the call from his lawyer, but he agreed to talk as long as he did not have to put anything in writing or sign anything without her okay. He made incriminating statements about the robbery, and he was eventually indicted for that crime as well. Prior to trial on the robbery charge, the defendant’s lawyer moved to suppress the arresting officer’s testimony about the defendant’s statements.

How should the court rule?

A

Deny the motion, because the defendant’s statements were made voluntarily after receiving Miranda warnings.

  • The defendant’s motion should be denied because his interrogation did not violate his Fifth Amendment right to counsel. At any time prior to or during interrogation, a suspect may invoke a Miranda (Fifth Amendment) right to counsel. However, the request must be unambiguous and specific. If the defendant agrees to answer questions orally, but requests the presence of counsel before making any written statements, the defendant’s oral statements are admissible. The defendant’s agreement to talk constitutes a voluntary and knowing waiver of the right to counsel, even if it could be argued that it indicates a misunderstanding of the evidentiary effect of oral statements. [Connecticut v. Barrett (1987)] T
53
Q

Crim Law & Procedure

On the last play of a playoff football game, a game-winning touchdown was nullified by a questionable penalty called by the referee. To register her displeasure but without intending to hit anyone, a fan sitting in the stands threw a bottle onto the field that just missed the head of the referee, who was looking in the other direction and did not see the bottle being thrown. The fan was charged with assault.

Should the fan be convicted?

A

No, because the referee did not see the bottle, nor did the fan intend to hit anyone.

  • The fan should not be convicted under either definition of assault. Criminal assault is either: (i) an attempt to commit a battery, or (ii) the intentional creation, other than by mere words, of a reasonable apprehension in the mind of the victim of imminent bodily harm. In the instant case, the fan did not intend to hit anyone. Thus, the fan’s actions do not constitute an attempt to commit a battery, which would require a specific intent to bring about bodily injury or an offensive touching. In addition, the fact that the referee did not see the bottle being thrown at him means that the defendant did not create in the referee a reasonable apprehension of imminent bodily harm. Consequently, the fan is not likely to be convicted of either type of assault.
54
Q

Crim Law & Pro

The defendant became very intoxicated one night. As he was staggering home, he came upon a construction site in which several large pieces of heavy equipment were parked. Having had heavy equipment training in the military, the defendant decided it would be fun to rearrange all the machines so that the operators would be very surprised when they returned to work the next day. He started up the largest piece of heavy equipment and drove it toward the edge of the site, but because he was so intoxicated, he lost control of it, and it rumbled out into the street, weaved along for about a quarter mile, and then crashed into a house, flattening it. In this jurisdiction, it is a misdemeanor to tamper with heavy equipment on a construction site. The defendant is prosecuted on the tampering charge, as well as for reckless damage of the house.

Should he be convicted of the reckless damage charge?

A

Yes, because he was intoxicated while driving a huge piece of earthmoving equipment.

  • The defendant should be convicted because he was intoxicated when he damaged the property. The defendant is being charged with reckless damage to property. A person acts recklessly when he consciously disregards a substantial or unjustifiable risk that a prohibited result will follow and this disregard constitutes a gross deviation from the standard of reasonable care. Driving earthmoving equipment while intoxicated would be considered to be reckless because of the great potential for destruction arising from the huge size and power of the equipment.
55
Q

Crim Law & Pro

A computer programmer sent a computer virus anonymously via e-mail to a business. The programmer believed that the virus would just disable the business’s e-mail program for a short time without causing any additional damage, although he was aware that it very infrequently caused widespread damage to the infected computer. However, because of a hidden bug in the business’s e-mail program, the virus infected the computer’s entire hard drive, eventually rendering it unusable. Not only did the business lose important data, it also had to replace the computer, at a cost of over $1,000. The jurisdiction in which this occurred has a modern criminal code patterned after the Model Penal Code. One of its statutes makes it a criminal offense to “knowingly cause over $200 in damage to another’s property.”

May the programmer be found guilty of violating the statute?

A

No, because the programmer did not know that the virus would cause damage to the computer’s hard drive.

  • The computer programmer cannot be found guilty of violating the statute because he did not know that his act would cause the damage to the business’s computer that it did. Under the Model Penal Code fault standards adopted by modern criminal codes, a person acts “knowingly” with respect to the nature of his conduct when he is aware that his conduct is of that nature or that certain circumstances exist. He acts knowingly with respect to the result of his conduct when he knows that his conduct will necessarily or very likely cause such a result. When a statute establishes a culpable state of mind without indicating to which material elements of the offense it is to apply, the statute will be interpreted as requiring that state of mind for every material element of the offense. In this case, the statute requires that the defendant “knowingly cause over $200 in damage to another’s property.” The requirement that the damage caused be over $200 is a material element of the offense because it defines the harmful result that will trigger criminal liability under the statute. Thus, the programmer must have known that his act of sending the computer virus would necessarily or very likely cause over $200 in damage to the business’s computer to be liable under the statute in this case.
56
Q

Crim Law & Pro

Based on a tip from a reliable informant that an attorney was illegally selling automatic weapons and ammunition from his storefront office, the police obtained a warrant to search for weapons at the office. When they arrived at the building, they saw a client exiting the attorney’s office and placing what appeared to be a weapon inside his jacket. The police stopped the client on the street and an officer patted down his outer clothing. The officer felt no weapon but did feel a bag with several small tube-shaped objects in it. She immediately placed the client under arrest. The contents of the bag were later determined to be marijuana cigarettes.

Prior to trial on the narcotics charge, the client sought to suppress introduction of the marijuana as evidence. The arresting officer testified at the suppression hearing that, based on her long experience as a narcotics officer, she concluded immediately that the bag contained marijuana cigarettes when she first touched it.

If the officer’s testimony is believed, how should the court rule on the motion to suppress the marijuana evidence?

A

Deny it, because the police had a reasonable suspicion that the client might be armed and dangerous.

  • The client’s motion should be denied because the seizure of the marijuana was properly within the scope of the stop and frisk. A police officer may stop a person without probable cause for arrest if she has an articulable and reasonable suspicion of criminal activity. [Terry v. Ohio (1968)] In such circumstances, if the officer reasonably believes that the person may be armed and dangerous, she may conduct a protective frisk. The scope of the frisk is limited to a patdown of the outer clothing for concealed instruments of assault, but the officer may reach into the suspect’s clothing and seize any item that the officer reasonably believes, based on its “plain feel,” is a weapon or contraband. [Minnesota v. Dickerson (1993)] Here, the officer believed that the client put a weapon in his jacket as he was leaving a place where weapons and ammunition were being sold illegally; thus, she had reasonable grounds to conduct both a stop and a frisk. If the court accepts the officer’s testimony that she instantly recognized the marijuana cigarettes based on the patdown only without any further conduct, they were properly seized and can be admitted into evidence.
57
Q

Crim Law & Pro

A man lost some money while playing poker with several people at his friend’s house. When the man accused his friend of cheating, his friend asked him to leave. The man became abusive and refused to leave, so his friend and a couple of other players forced him to go. Angry and determined to get his money back, the man went to his home and picked up his gun. He headed back to his friend’s house, intending to shoot his friend if he did not give back the money. However, due to the altercation at the house, his friend had called the police. Just as the man was about to step onto his friend’s property, the police pulled up and stopped him. They frisked him, found the gun in his pocket, and arrested him. A state statute prohibits entry onto the property of another with the intent to commit violence thereon.

If charged with attempt under this statute, will the man be found guilty?

A

Yes, because the man was trying to enter the property and he had the necessary state of mind.

  • The man will likely be found guilty. An attempt requires both a specific intent to commit the crime and an overt act in furtherance of that intent. Given that the man intended to enter his friend’s property and was apprehended just before doing so, both requirements for attempt can be established.
58
Q

Crim Law & Pro

A state statute allows for criminal trials by a jury composed of six jurors. Five of the six jurors must concur for a guilty verdict. The defendant is charged with petty larceny, which carries a maximum sentence of one year’s imprisonment, plus a fine of $2,500. Before voir dire begins, the defendant objects to both the six-member jury and the fact that only five of six jurors are needed for a conviction.

Should the trial judge overrule the objection?

A

No, because the number of jurors needed for a conviction is unconstitutional for this offense.

  • The judge should not overrule the objection. There is no constitutional right to a jury of 12, but there must be at least six jurors to satisfy the right to a jury trial under the Sixth and Fourteenth Amendments. Also, the Supreme Court has held that juries must be unanimous.
59
Q

Crim Law & Pro

A state statute has adopted the common law definition of larceny. Another statute provided as follows:

“It shall be an affirmative defense to a crime if the defendant establishes by clear and convincing evidence that, due to a mental disease or defect, he was unable to appreciate the criminality of his conduct or conform his conduct to the requirements of the law.”

A homeowner was leaving town for two weeks and he asked his neighbor to stop by the house each day and water the plants. While at the homeowner’s home, the neighbor found the keys to the homeowner’s new car. The neighbor took the car and drove it into town to show his friends. The neighbor told all of his friends that he had purchased the car. The homeowner returned home three days early, saw that the car was missing, and called the police. Later that day, the neighbor was arrested and charged with larceny.

At the neighbor’s trial, the neighbor testified that he intended to return the car. Additionally, two psychiatrists testified that, due to a mental defect, the neighbor suffered from an extreme inferiority complex and delusions of grandeur. The doctors further testified that his mental condition caused him to take the car and to tell other people that he owned it. At the conclusion of the evidence, the court’s instructions to the jury included the following:

  1. If you find by a preponderance of the evidence that the defendant intended to return the car, you should find the defendant not guilty.
  2. If you find by a preponderance of the evidence that, due to a mental disease or defect, the defendant was unable to appreciate the criminality of his conduct or conform his conduct to the requirements of the law, you should find the defendant not guilty.

The neighbor was found guilty and he appealed, claiming that the jury instructions violated his constitutional rights.

How should the appellate court rule?

A

Instruction 1 was unconstitutional; Instruction 2 was constitutional.

  • The appellate court should rule that only Instruction 2 was constitutional. Due process of law requires a state to prove each element of the crime charged beyond a reasonable doubt. However, as to affirmative defenses to the criminal charge, the Supreme Court has held that the state can place the burden of proof on the defendant without violating the defendant’s constitutional rights. [Leland v. Oregon (1952)] Common law larceny is the taking and carrying away of property in the possession of another with the intent to permanently deprive the other of the property. Since the intent to permanently deprive is an element of the crime, the state cannot require the defendant to prove that he intended to return the car, which would negate the required intent for larceny. The state must prove beyond a reasonable doubt that he intended to permanently deprive. Hence, Instruction 1 is unconstitutional.
60
Q

Crim Law & Pro

An employee worked as a third-shift supervisor at a manufacturing plant. One of his duties was to ensure that all timekeeping records accurately reflected the time his crew actually worked. Workers, including the employee, were then paid for whatever hours the timecards reflected. The employee was also required to assist in submitting budgets for payroll. Needing to leave work early for a second job that he obtained, the employee had one of his trusted co-workers punch his card out at the regular time every day of the week. At the end of the week, he signed the timecard with those hours included, and was paid accordingly. He continued to do this for several weeks before being discovered.

What crime has the employee committed?

A

False pretenses.

  • At common law, theft by false pretenses occurs when a defendant (i) obtains title; (ii) to the property of another; (iii) by an intentional (or knowing) false statement of past or existing fact; (iv) with the intent to defraud another. In the instant case, the employee’s conduct meets all of the elements of the crime. The employee obtained title to the property by falsely misrepresenting the number of hours that he worked, with the intent that the company would pay him for the hours. Thus, he has committed the crime of false pretenses.
61
Q

Crim Law & Pro

A homeowner decided to destroy his home by fire to collect the insurance money. To work up his courage, he had several drinks at a local bar. When he returned to his block that night, he was so intoxicated that he mistakenly believed that his neighbor’s house, which was 20 feet to the right of his house and looked very similar, was his own house. He started a fire under the back porch and went off a short distance to watch it burn. Suddenly he realized that he had the wrong house. He ran back and grabbed a garden hose and was able to put out the fire with just some slight charring of the porch.

If the homeowner is charged with arson in a jurisdiction retaining the common law rules, what is the likely verdict?

A

Guilty, because he acted with malice.

  • The homeowner should be found guilty. Common law arson consists of the malicious burning of the dwelling of another. At common law, the state of mind required-malice-is satisfied not only by intentionally burning the dwelling of another but also by acting with reckless disregard of an obvious risk that the structure would burn. While many courts ordinarily require that the defendant be subjectively aware of the risk, they will find malice when the failure to be aware of the risk is due to voluntary intoxication. Even had the homeowner done what he intended, he would have put his neighbor’s house in jeopardy of burning. The fact that his intoxication caused him to fail to recognize the risk would not be a defense. Nor could he raise a mistake of fact defense because mistake of fact must be reasonable to negate the existence of malice, and here the facts state that his mistake was caused by his intoxication.
62
Q

Crim Law & Pro

A factory foreman was suspected of having murdered, for pay, the rival of a local union leader. After the police arrested the foreman at his home and he was taken to the police station, the officers who remained at the house asked the foreman’s aunt, who was visiting him for the week, if she knew where any firearms could be found in the house. She went into the bedroom and returned with a pistol. Ballistics experts established that the pistol had been used to murder the victim, and the foreman’s fingerprints were all over the pistol. At a subsequent grand jury proceeding, the district attorney introduced the pistol and the related ballistics and fingerprint evidence, and the grand jury indicted the foreman.

If the foreman seeks to quash the indictment, will he prevail?

A

No, because the evidence was offered before a grand jury, not a court.

  • A grand jury may consider any evidence available to it in determining whether probable cause exists to return an indictment against the defendant. Because the exclusionary rule does not apply, a grand jury may base its indictment on evidence that would not be admissible at trial. Thus, even if the pistol was the product of an illegal search and seizure, and the grand jury based its indictment on this evidence, the foreman will not prevail in his attempt to quash the indictment.
63
Q

Crim Law & Pro

The defendant bought a new bow and arrow set at a local sporting goods store and went to a public park to try it out, even though he knew that practicing his marksmanship at the park was a violation of park regulations and constituted a misdemeanor. Right at the moment that the defendant fired his first arrow, a park ranger yelled at him from a distance to “stop shooting, stupid.” Perturbed that he was caught so early, the defendant decided to fire an arrow a couple of feet above the ranger’s head. Unfortunately, the defendant’s aim was slightly off, and the arrow struck the ranger right between the eyes, killing him instantly.

The defendant is charged with homicide for the park ranger’s death. At trial, the jury was given instructions on common law murder and manslaughter.

The crimes below are listed in descending order of seriousness.

If the jury believes the defendant’s testimony that he did not intend to hit the park ranger with the arrow, what is the most serious charge for which the jury may find him guilty?

A

Murder.

  • Even if the jury believes that the defendant did not intend to hit the park ranger with the arrow, the defendant nonetheless may be found guilty of murder. At common law, murder is the unlawful killing of a human being with malice aforethought. Malice aforethought exists if the defendant has any of the following states of mind: (i) intent to kill; (ii) intent to inflict great bodily injury; (iii) reckless indifference to an unjustifiably high risk to human life (i.e., an “abandoned and malignant” heart); or (iv) intent to commit a felony. Here, a jury could find that the act of shooting an arrow toward the close proximity of another person constituted a reckless indifference to a high risk of serious injury or death.
64
Q

Crim Law & Pro

An undercover agent for a federal drug enforcement agency informed a state law enforcement agency that a large amount of cocaine was being mailed to a resident of that state. The cocaine would be mailed in a large box and wrapped distinctively. The agent further informed the agency that the resident was not the purchaser of the cocaine, but was only acting as an intermediary. The cocaine would be picked up within a few days by the buyer, who was from a neighboring state. The agency immediately placed the resident’s house under surveillance. In a few days, a large box wrapped as the undercover agent described was delivered by the post office. The agency did not make an arrest, but kept the house under surveillance. Two days later, a man driving a car with plates from the neighboring state arrived at the house. He entered the house and came back out shortly thereafter carrying what appeared to be the same box. The suspect placed the box in the trunk of his car and drove off. Two blocks later, the car was stopped, the suspect arrested, and officers for the agency searched the entire vehicle, acting without a warrant. The box in the trunk was opened and cocaine was found. The suspect was charged with possession of cocaine. At a pretrial hearing, he moved to suppress the cocaine.

How should the court rule on the motion?

A

Deny it, because the officers had probable cause to search the trunk.

  • The suspect’s motion should be denied. As a general matter, to conduct a constitutionally valid search, the police must have a search warrant based on probable cause unless the case falls under one of the exceptions to the warrant requirement. One well-established exception to the warrant requirement is the automobile exception. If the police have probable cause to believe that an automobile contains contraband or evidence of a crime, they may search whatever area of the car that may contain the object of their search without having to get a warrant. Probable cause to search is defined as reasonable grounds for believing that a particular item of seizure is located at a particular place. From the facts given, the officers had reasonable grounds to believe that cocaine was in the trunk of the car.
65
Q

Crim Law & Pro

A police officer witnessed a bar patron exit a bar with an open bottle in his hand, get into a car, and turn the wrong way from the bar’s parking lot onto a one-way street. The officer immediately turned on his siren and pursued the car for a couple of miles. During that pursuit, the car repeatedly weaved in and out of its lane of traffic. Eventually, the car pulled over, and the officer placed the driver under arrest for drunk driving. After handcuffing the driver and placing him in the back seat of his squad car, the officer looked under a blanket lying on the floor of the car’s passenger compartment. Under the blanket, he found an open bottle of beer. Before his trial on charges of drunk driving and driving with an open container of alcohol in the car, the defendant moves to suppress from evidence the open bottle of beer.

Should the motion be granted?

A

No, because the officer had reason to believe that the car contained evidence of the crime for which the defendant was arrested.

  • The driver’s motion should be denied. As a general matter, to conduct a constitutionally valid search, the police must have a search warrant based on probable cause unless the case falls under one of the exceptions to the warrant requirement. One exception to this rule is applicable here. After arresting a person who was recently in an automobile, the police may search the passenger compartment of the automobile if they reasonably believe that evidence of the offense for which the person was arrested may be found in the automobile. That is the case here, as the officer arrested the driver for drunk driving and saw the driver leave a bar with an open bottle in his hand and get into the automobile.
66
Q

Crim Law & Pro

A defendant was charged with robbery and felony murder based on a death that arose during the robbery. The defendant pleaded not guilty and insisted on a jury trial. Right before the trial began, he fired his attorney and decided to defend himself. The court made a finding that the defendant was competent to represent himself at trial. The defendant then insisted on trying both of his charges separately in two different trials. The trial judge asked the defendant if he was confident that he wanted to have two separate trials. The defendant replied: “I am, your Honor.” The felony murder case was tried first, and the jury found the defendant not guilty. The defendant then moved to dismiss the robbery charge based on double jeopardy.

How should the court rule on the defendant’s motion?

A

Deny the motion, because the charges could have been tried together but the defendant consented to the separate trials.

  • The court should deny the motion. Where charges can be tried in a single trial, but the defendant consents to having two separate trials, there is no double jeopardy violation. Just as consent is an exception to the warrant requirement, consent to a second trial is an exception to the double jeopardy rule.
67
Q

Crim Law & Pro

A gun collector ordered a rifle from a gun catalog. The rifle had a barrel 16 inches long and a pistol-type grip instead of the more usual rifle stock, so that the entire weapon was only 22 inches long. The collector was aware of a state penal statute that prohibited the possession of “any sawed-off shotgun or rifle.” He was also aware that another statute defined sawed-off shotgun or rifle so as to include any such weapon whose barrel was less than 16 inches in length. He was unaware that the same statute also included in its definition of the prohibited weapons any shotgun or rifle whose overall length was less than 24 inches. When the rifle he had ordered arrived in the mail, he carefully measured it to confirm that its barrel was exactly 16 inches in length.

While driving to the target range one day, the collector was stopped for having a defective taillight, and the traffic officer saw the rifle he had ordered lying on the back seat of his car in plain view. The collector was arrested and subsequently prosecuted for possession of a sawed-off rifle.

What will be the probable outcome of the trial?

A

He will be convicted, because his reasonable investigation does not vitiate violation of the statute arising from a mistake of law.

  • The reasonableness of the collector’s belief that the length of his rifle did not violate the statute provides no defense to the crime charged. Unless the collector can establish a valid defense, he will be liable for violating the statute because he possessed a rifle less than 24 inches long. It is not a defense to a crime that the defendant was unaware that his acts were prohibited by the criminal law or that he mistakenly believed that his acts were not prohibited, even if such ignorance or mistake was reasonable. Here, the collector knew that the rifle he was purchasing was 22 inches long, but he was unaware that the statute prohibited possession of a rifle whose length was less than 24 inches. Such ignorance of the prohibition of the statute will not constitute a defense to the charge of possession of a sawed-off rifle, even if such ignorance remained after a reasonable investigation by the collector.
68
Q

Crim Law & Pro

Right before the beginning of the defendant’s trial for arson, a bailiff approached the defendant and got him to admit that he had burned down the house in question. When the trial began, the defendant testified that he had nothing to do with the fire in question. In rebuttal, the prosecution seeks to put the bailiff on the stand to testify as to the defendant’s statements, but the defendant’s attorney objects.

How should the court rule regarding the objection?

A

Overruled, because the prosecution is seeking only to impeach the defendant’s testimony.

  • This question illustrates the operation of the Sixth Amendment right to counsel approach, which you should use to evaluate the admissibility of any statements made after the defendant has been charged with the relevant crime. The Sixth Amendment provides defendants with a right to counsel at any post-indictment interrogation. Since the defendant was on trial for arson, any interrogation relating to those charges must take place, if at all, in the presence of the defendant’s counsel unless the defendant has knowingly and voluntarily waived the right.
69
Q

Crim Law & Pro

The victim was walking out of a store when she saw someone suddenly fall to the street suffering an apparent heart attack. However, that person was in fact an accomplice of the defendant. With the victim’s attention momentarily diverted, the defendant removed the victim’s wallet from her purse. Another passerby shouted to the victim, who turned and caught the defendant by his sleeve. The defendant pushed her hand away and started to run, but tripped over a curb and dropped the wallet when he fell, and the victim was able to recover it.

The crimes below are listed in descending order of seriousness.

What is the most serious crime for which the defendant could be convicted?

A

Robbery.

  • The defendant could be convicted of robbery under these facts because of his use of force. Robbery is larceny from the person or presence of the victim by means of violence or intimidation. Larceny is the taking and carrying away of the tangible personal property of another by trespass with the intent to permanently deprive the person of his interest in the property. The use of force constituting a battery is sufficient for robbery, and the perpetrator’s use of force to overcome the person’s resistance to the taking is also sufficient force to constitute a robbery. All that is required for taking is that the perpetrator have possession of the item, and the slightest movement will suffice for the carrying away. The fact that the defendant subsequently dropped the wallet does not negate his completion of the crime.
70
Q

Evidence

A defendant was charged with murdering his boss. After obtaining a valid search warrant and executing a valid search of the defendant’s office, an officer found a love letter from the defendant’s wife to his boss describing their sexual relations. The letter stated, “I can no longer hide my love for you from my husband. I intend to tell him about us and leave him for you.” At trial, the officer seeks to testify about the contents of the letter as proof of the defendant’s motive for killing his boss.

The defense counsel should object on which of the following grounds?

A

The testimony violates the best evidence rule.

  • The defense counsel should object because the officer’s testimony violates the best evidence rule. Under the best evidence rule, the original writing must be produced when proving the material terms of that writing. Secondary evidence of the writing, such as oral testimony regarding the writing’s contents, is permitted only after it has been shown that the original is unavailable for some reason other than by serious misconduct of the proponent. Here, the terms of the letter are material and no justification has been given for why it cannot be produced. Thus, the original letter must be produced and the officer’s testimony violates the best evidence rule.
71
Q

Evidence

The plaintiff was driving her daughter to school when their car was struck broadside by a car driven by the defendant at an intersection controlled in all directions by stop signs. The plaintiff and her daughter were taken by ambulance to the hospital. In a personal injury action brought by the plaintiff and her daughter against the defendant, pretrial discovery revealed that both cars were in perfect mechanical condition just before the accident, and the defendant was on his way home from work at the time of the accident, but had stopped off at a bar before he reached the intersection at which he struck the plaintiff’s car. There is no witness available to testify as to how much the defendant had to drink at the bar that day.

At trial, the plaintiff calls a co-worker of the defendant, who testifies over objection that the defendant has a reputation as a hard drinker who tolerates alcohol well but who always drinks a great deal at any one drinking occasion, as witnessed by the co-worker at numerous company events.

Was it error for the trial court to admit his testimony?

A

Yes, because the plaintiff may not attempt to prove that the defendant acted in a particular way on one occasion in conformity with his reputation as to that behavior.

  • It was error to admit the reputation evidence from the co-worker because, in civil trials, character evidence is inadmissible to prove that the litigant acted in conformity with that character. An exception exists when the litigant’s character is directly in issue (e.g., in a defamation action), but that is not the case here.
72
Q

Evidence

The defendant is charged with having been one of two men who robbed a tavern and its patrons at gunpoint at 5:30 p.m. on December 16.

The defendant calls a witness to testify that he was at the defendant’s house at about 9:30 a.m. on December 16, and that as he was leaving, the defendant said to him, “I’m going to my mother-in-law’s house this afternoon for a birthday party.”

Is the witness’s testimony admissible?

A

Yes, it is hearsay within an exception, and thus admissible.

  • The evidence is admissible because it is a declaration of the defendant’s present state of mind, offered as circumstantial evidence that he carried out his intent to go to his mother-in-law’s house. Hearsay is a statement, other than one made by the declarant while testifying at the trial or hearing, offered in evidence to prove the truth of the matter asserted. One exception to the hearsay rule is for statements of present state of mind. Declarations of an existing state of mind are admissible not only when the declarant’s state of mind is directly in issue and material to the controversy, but also when the declarant’s state of mind is not directly in issue, but the declarations of intent are offered to show subsequent acts of the declarant; i.e., a declaration of intent to do something in the future is offered as circumstantial evidence tending to show that the intent was carried out. The defendant’s statement that he was going to his mother-in-law’s house is a statement made by the declarant out of court. This statement is offered to prove the truth of the matter asserted therein: that on the day of the alleged armed robbery, the defendant intended to go to his mother-in-law’s house. This is being offered as circumstantial evidence that he did go to his mother-in-law’s house. Thus, the statement is hearsay. However, the statement does come within the present state of mind exception. Although the defendant’s state of mind is not directly in issue, his statement is a declaration of intent to do something offered to show that such intent was in fact carried out. Therefore, the statement is admissible under the present state of mind exception to the hearsay rule. [See Mutual Life Insurance Co. v. Hillmon (1892)]
73
Q

Evidence

A plumbing contractor sued a homeowner, alleging that the homeowner refused to pay for extensive pipe repairs performed on her home by an employee of the contractor. The contractor called the employee to the stand as a witness. The employee, under oath, testified that he did not perform any work at the homeowner’s home. The employee also denied writing a letter to a friend telling the friend that he was going to do plumbing work on the homeowner’s house. Without releasing the employee as a witness, the contractor offers in evidence the letter written by the employee to his friend.

Which of the following is NOT a proper basis for admitting the employee’s letter?

A: Testimony by the employee’s wife that she recognizes the employee’s handwriting.
B: The friend’s testimony that the statements in the letter are responsive to a prior letter from the friend to the employee.
C: Comparison by the jury of the letter with another letter that the employee has admitted writing.
D: In-court comparison by the friend, a nonexpert, of the letter with another letter that the employee has admitted writing.

A

D: In-court comparison by the friend, a nonexpert, of the letter with another letter that the employee has admitted writing.

  • The friend’s comparison of the letters is not a proper basis for authenticating the employee’s letter. Before a writing may be received into evidence, the writing must be authenticated by proof showing that the writing is what the proponent claims it is. The Federal Rules list several examples of proper methods of authentication through evidence of the genuineness of the handwriting of a letter writer. None of them, however, would permit an in-court comparison by the friend of the contested letter with another letter established to be the employee’s. Under Federal Rule of Evidence 901(b), an expert witness or the trier of fact can determine the genuineness of a writing by comparing the questioned writing with another writing proved to be genuine. Thus, (C) would be a proper basis for admitting the employee’s letter, because the jury, as the trier of fact, can compare the letter with another letter that the employee has admitted writing. The friend, however, cannot undertake that comparison because he is not an expert witness. While Rule 901 does not limit the methods of authentication, the rule governing opinion testimony by lay witnesses [Fed. R. Evid. 701] would preclude the friend’s in-court comparison. To be admissible, opinion testimony by lay witnesses must be (i) rationally based on the perception of the witness; (ii) helpful to a clear understanding of his testimony or to determination of a fact in issue; and (iii) not based on scientific, technical, or other specialized knowledge. Here, the friend can add nothing to the jury’s determination of the authenticity of the letter because the jury can compare for itself that letter with a letter the employee has admitted writing. Because the friend’s in-court comparison would not be helpful to the jury here, it would not be a proper basis for authenticating the employee’s letter. (A) is incorrect because a lay witness who has personal knowledge of the handwriting of the supposed writer may state her opinion as to whether the document is in that person’s handwriting. Therefore, the employee’s wife could properly authenticate the letter by testifying that she recognizes his handwriting. (B) is incorrect because the friend’s testimony is proper circumstantial evidence of authentication under the reply letter doctrine. Under this doctrine, a letter may be authenticated by evidence that it was written in response to a communication sent to the claimed author. The content of the letter must make it unlikely that it was written by anyone other than the claimed author of the writing. Here, the employee’s letter to the friend can be properly authenticated by the friend’s testimony that it was a reply to a letter that he sent to the employee.
74
Q

Evidence

The defendant is on trial for shoplifting. As part of his defense, the defendant calls to the stand a restaurant cashier, who will testify that the defendant is a regular customer and has corrected an undercharge on her bill several times.

Is the testimony of the cashier admissible?

A

No, because the evidence is not a proper means for proving good character.

  • The cashier’s testimony is not admissible to prove good character. Generally, a criminal defendant can introduce evidence of her good character to show that she did not commit the crime charged. This is accomplished through the reputation or opinion testimony of a witness. The testimony must relate to the trait involved in the case and the witness cannot testify to specific acts of conduct. Here, the defendant is entitled to call the restaurant cashier to the stand to testify to her good character, but she can do so only by giving her personal opinion or by testifying to the defendant’s reputation concerning honesty (the trait involved in a shoplifting case). The cashier’s testimony in this case is impermissible because she is attempting to testify to the defendant’s history of correcting undercharges on her bill, which are specific acts of conduct. Thus, her testimony is inadmissible.
75
Q

Evidence

At the trial of the plaintiff’s personal injury action against the defendant, a pedestrian, who was near the accident scene but did not see what happened, testifies that an eyewitness to the accident shouted, “Good Lord! The green car just ran through a red light and hit the red car!” Previous evidence had established that the defendant drove a green car and the plaintiff a red one. The defendant offers to call to the stand the brother of the eyewitness, who will testify that he spoke with the eyewitness the day after the accident, and he said that the light was green when the green car drove through the intersection. The eyewitness had moved to a foreign country prior to trial.

Should this evidence be admitted over the plaintiff’s objection?

A

Yes, for the purpose of impeachment only.

  • The evidence is admissible for impeachment purposes as an inconsistent statement of a hearsay declarant. Because the credibility of a hearsay declarant is as much at issue as the credibility of an in-court witness, Federal Rule 806 allows statements of a hearsay declarant to be impeached to the same extent as those of an in-court witness. Thus, a statement of the declarant made at any time that is inconsistent with his hearsay statement may be offered into evidence for impeachment purposes. Here, the eyewitness’s hearsay statement (which probably qualified as an excited utterance) was testified to by the pedestrian. The eyewitness’s subsequent statement to his brother is inconsistent with his hearsay statement and is therefore admissible to discredit that statement.
76
Q

Evidence

The plaintiff sued the defendant, the owner of an art gallery, alleging that the defendant charged him a price higher than what was originally quoted to him for the purchase of a rare sculpture. During the plaintiff’s testimony, he stated that he purchased the sculpture from the gallery on a particular date and then realized two days later that his credit card was charged in an amount over that which he was originally quoted by the defendant. During its defense, the defendant presented the testimony of the art gallery’s clerk, who testifies that she remembers the plaintiff coming into the gallery and purchasing the sculpture a week before the date testified to by him, because he signed the purchase order with such an unusual signature.

If the plaintiff objects to this testimony, should the trial court admit it?

A

No, because the date of purchase is a collateral matter.

  • Testimony as to the date of purchase of the sculpture should not be admitted because its minimal relevance is substantially outweighed by considerations of waste of time and confusion of the issues under Rule 403. Whether the plaintiff actually purchased the sculpture one week sooner than the date testified to by him has no bearing on the amount he should have been charged, which is the issue in controversy. The only relevance of the clerk’s testimony is to cast doubt on the plaintiff’s credibility, but it is not admissible for impeachment purposes either. When a witness makes a statement not directly relevant to the issues in the case, the rule against impeachment on a collateral matter bars his opponent from proving the statement untrue either by extrinsic evidence or by a prior inconsistent statement. As noted previously, the plaintiff’s statement as to the date on which he purchased the sculpture is not directly relevant to any other issue in the case. Thus, the defendant is not permitted to prove the statement untrue by means of the clerk’s testimony that the plaintiff made the purchase on a different day.
77
Q

Evidence

In January of 2010, the defendant proposed to his girlfriend. During the engagement, the defendant confided in her about various drug deals in which he was participating. The woman swore that she would never reveal any of his confidences. On January 1, 2011, the couple married. The defendant continued to share with his wife information concerning his illegal drug activity. The wife’s only rule was that he could not participate in any illegal drug transactions in their home. On one occasion in 2011, the wife came home unexpectedly and saw the defendant completing a drug transaction in the living room. The defendant was not aware that his wife had observed the event. In 2013, the defendant was charged with 57 counts of illegal drug sales that occurred between 2009 and 2012. The prosecutor wishes to call the defendant’s wife as a witness for the state.

Assuming that the defendant’s attorney makes appropriate objections, which of the following statements is correct regarding testimony by the defendant’s wife?

A: She can testify about the defendant’s 2011 statements if she desires.
B: She must testify to the defendant’s 2010 statements.
C: The defendant can keep her from testifying about his 2010 statements.
D: She can testify to the drug sale that she observed in 2011 if she desires.

A

D: She can testify to the drug sale that she observed in 2011 if she desires.

  • The defendant’s wife can testify about the drug sale in 2011 if she wants to. The federal courts recognize two separate and distinct privileges related to marriage. First, in a criminal case, a spouse can testify if she wants to testify, but the spouse cannot be compelled to testify. The privilege applies to all information that the spouse has gained before or during the marriage. This privilege ends when the marriage ends. Second, in any type of case, a spouse can refuse to disclose, or prevent a spouse from disclosing, confidential communications made between the spouses during the marriage. While the defendant can object to his wife’s testimony concerning confidential communications during the marriage, he has no privilege to object to other testimony by her. It is clear that the 2011 drug transaction was not a confidential communication, since the defendant was not aware of her presence.
78
Q

Evidence

The plaintiff sued the defendant dry cleaner, claiming that it had permanently ruined her $10,000 mink coat by cleaning it with a solvent that left an extremely offensive odor that smelled like “skunk.” Further attempts to have the odor removed by other cleaning services were unsuccessful. The odor was so bad that she could no longer wear the coat.

At the trial, the plaintiff testified to the above facts. She then identified a mink coat as her coat that the defendant had ruined. She testified that it still smelled the same as it did after the defendant had cleaned it. The plaintiff’s counsel offered to introduce the coat for the purpose of having the jury smell it. Defense counsel objected.

How should the court rule?

A

The coat is admissible based on the plaintiff’s testimony.

  • The court should admit the coat based on the plaintiff’s testimony. Federal Rule of Evidence 402 provides that all relevant evidence is admissible unless a specific rule keeps the evidence out or limits its admissibility. In the case of real evidence, the object at issue is presented for inspection by the trier of fact. Such evidence can be presented to any of the senses of the jury from which the jury can obtain relevant information. Clearly the odor of the coat is a central issue in the case and the jury would obtain relevant evidence on that issue by smelling the coat. Admitting the coat for the stated purpose would violate no other rules of evidence.
79
Q

Evidence

On an icy day, a vehicle driven by the defendant struck the plaintiff’s car in the rear, smashing a taillight and denting the plaintiff’s bumper. Before the plaintiff could say anything, the defendant rushed out of his car and told the plaintiff, “Look, if you’ll take $500 for the damage, I’m sure my insurance company will pay for it.” The plaintiff refused and sued the defendant for damage to his car and minor personal injuries. The plaintiff wishes to testify as to the defendant’s statement at the time of the accident. The defendant objects.

Should the court allow the defendant’s statement to be admitted?

A

Yes, because it is a statement by an opposing party.

  • The statement by the defendant, who is one of the parties to the action, is admissible as a statement by an opposing party (commonly called an admission). Federal Rule 801(d)(2) provides that a statement offered against a party that is the party’s own statement is not hearsay and therefore cannot be excluded by the rule against hearsay. Assuming that it is relevant and not barred by other rules, the statement is admissible. Here, the defendant’s statement is being offered against him at trial. It is relevant because it can be interpreted as a prior acknowledgment by the defendant that he was not totally blameless in the accident, which is undoubtedly inconsistent with his contentions at trial. The statement does not violate Federal Rule 408, which makes offers to compromise a disputed claim inadmissible to prove or disprove the validity or amount of a disputed claim, because it was made by the defendant before the plaintiff made any claim; i.e., there was not yet an actual dispute between the parties. Nor does the statement violate Rule 411, which bars evidence that a person has liability insurance when offered to show fault or ability to pay, because the defendant’s reference to his insurance was an intrinsic part of his admission and could not be readily severed from it. No other rules barring relevant evidence apply, so the statement should be admitted.
80
Q

Evidence

As a result of an automobile accident at an intersection, the plaintiff sued the defendant, claiming that the defendant’s car was traveling at a high rate of speed and went through a red light just before the crash. A witness for the plaintiff testified that he observed the accident and that the plaintiff’s car was traveling at a low speed with a green light at the time of the accident.

Which of the following will the court find NOT admissible to admit to impeach the credibility of the witness?

A: A certified copy of a certificate of conviction for felony assault and battery seven years ago.
B: The testimony of the witness’s friend that, last month, while having a drink at a bar, the witness told her that the plaintiff’s light was red.
C: A record of an arrest one week ago for embezzlement.
D: On cross-examination of the witness, the question, “Isn’t it a fact that you lied to your employer last year concerning your meal expenses on a business trip?”

A

C: A record of an arrest one week ago for embezzlement.

  • A record of an arrest, even for a crime such as embezzlement, cannot be used to impeach the credibility of a witness. Since there was no criminal conviction, this would be classified as evidence of a prior bad act that demonstrates dishonesty. Federal Rule of Evidence 608 would allow an inquiry into such a prior bad act during cross-examination of the witness being impeached, but extrinsic evidence of such acts is not allowed under Rule 608, even if the witness denies the act on cross-examination. (A) might be admissible to impeach. Under Federal Rule 609, a prior felony conviction for crimes that do not involve dishonesty can be used to impeach, although the trial judge has discretion to exclude the evidence if its probative value is substantially outweighed by unfair prejudice or other Rule 403 considerations. (B) represents an acceptable method of impeachment. Prior inconsistent statements can be introduced to show that the witness’s testimony is not credible. (D) is an acceptable impeachment method. Under Rule 608, a witness may be interrogated on cross-examination with respect to any act of misconduct that is probative of truthfulness (i.e., demonstrates dishonesty). Lying on an expense report would be such an act.
81
Q

Real Property

Owen, the owner of Greenacre, sold it to Alice for $100,000. Alice did not record the deed, and left the country on an extended trip. Owen, seeing an opportunity to make a quick profit, partitioned Greenacre and sold the front half, Frontacre, to Bert in exchange for $50,000. Bert, who knew nothing about Alice’s interest in the property, promptly recorded his interest. Two months later, Bert sold Frontacre to Carl in exchange for $55,000. Carl was aware of Alice’s interest in Frontacre but recorded his deed to Frontacre anyway. Meanwhile, Owen executed a mortgage on the back half of the property, Backacre, to Bank in the amount of $40,000. Bank knew nothing of Owen’s transaction with Alice but neglected to record its mortgage interest. Six months later, Alice returned home and recorded her deed to Greenacre.

A statute in the jurisdiction provides: “Any conveyance of an estate in land, other than a lease for less than one year, shall not be valid against any subsequent purchaser for value, without notice, unless the conveyance is recorded.”

If Alice brings an action to quiet title in Greenacre, how is a court likely to classify her claim?

A

As superior to Owen’s rights in Backacre, inferior to Carl’s rights in Frontacre, and subject to Bank’s mortgage on Backacre.

  • Alice has title to Backacre subject to Bank’s mortgage, and Alice has no rights in Frontacre. The recording statute in the question is a pure notice statute, which allows subsequent purchasers for value and without notice of a prior conveyance to prevail over the prior transferee, regardless of whether the subsequent purchaser records. In addition, the “shelter rule” allows a person who takes from a bona fide purchaser to prevail against any interest that the transferor-bona fide purchaser would have prevailed against, even if the transferee had actual knowledge of the prior unrecorded interest. Thus, Carl would prevail over Alice even though Carl was aware of Alice’s interest in the parcel, because Carl obtained title from Bert, a bona fide purchaser. Thus, Alice’s rights in Frontacre are extinguished. With regard to Backacre, Alice has superior title to it over Owen, but takes subject to the mortgage on it by Bank because mortgagees for value are treated as “purchasers” under recording statutes and because Bank had no notice of Alice’s interest.
82
Q

Real Property

A homeowner borrowed $50,000 from a bank, secured by a mortgage on his home. Shortly thereafter, the homeowner sold his home to a buyer for $70,000 by a deed containing a recital signed by both parties that title passed “subject to” the bank’s mortgage, “which obligation grantee expressly assumes.” The buyer paid the homeowner $20,000, took possession of the house, and began making monthly payments of principal and interest to the bank. A few years later, a chemical manufacturing firm built a huge sulfur processing plant just down the road from the home, which caused the house to immediately decline in value to $35,000. Subsequently, the buyer stopped making the monthly payments to the bank. The bank exercised its contractual right of nonjudicial foreclosure and sold the house at a public auction for $34,000. The bank then brought suit against the homeowner and the buyer for $14,000, the difference between the proceeds of the foreclosure sale and the $48,000 principal remaining due on the original loan to the homeowner. The jurisdiction does not bar deficiency judgments.

Against whom should the bank be granted a judgment for $14,000?

A

Both the homeowner and the buyer.

  • Both the homeowner and the buyer are liable for the deficiency. If a sale of foreclosed property does not bring enough to satisfy the mortgage debt, the mortgagee/lender can bring a personal action against the mortgagor/debtor for the deficiency (as long as the jurisdiction does not bar deficiency judgments). When the mortgagor sells the mortgaged property and gives a deed, the grantee takes subject to the mortgage, which remains on the land. If the grantee does not sign an agreement to assume the mortgage, he does not become personally liable on the loan, and the original mortgagor remains personally liable. If the grantee does sign an assumption agreement, however, the lender is considered a third-party beneficiary of the agreement, and hence may recover from the assuming grantee, who is primarily liable, or the original mortgagor, who is secondarily liable. Here, the buyer signed the recital providing for the assumption, so she will be personally liable on the loan.
83
Q

Real Property

In January, an owner executed and delivered a mortgage on her property to a bank to secure a $50,000 loan. Due to a clerical error, the mortgage was not recorded at that time. On February 15, the owner entered into a contract to sell the property to a buyer for $150,000. On February 16, the owner took out a $30,000 mortgage on the property with a finance company. The finance company promptly and properly recorded its mortgage. Knowing nothing about either of the mortgages, the buyer closed on the property on April 1, tendering $150,000 to the owner. The owner gave the buyer a warranty deed to the property. On April 3, the bank discovered its error and properly recorded its mortgage that same day. The buyer recorded his deed to the property on April 6.

The jurisdiction in which the property is located permits mortgages on property under contract, and has a statute that provides: “No conveyance or mortgage of real property shall be valid against a subsequent purchaser for value and without notice whose conveyance is first recorded.” The bank brings an appropriate action to determine the status of its mortgage on the property.

What should be the court’s determination?

A

The buyer holds the property subject to both mortgages, and the bank’s mortgage is subordinate to the finance company’s mortgage.

  • The buyer holds the property subject to both mortgages, and the bank’s mortgage is subordinate to the finance company’s mortgage. The jurisdiction’s recording act is a race-notice statute. Under this statute, a bona fide purchaser is protected only if he records before the prior transferee or mortgagee records. Here, the buyer had record notice of the finance company’s mortgage, so he was not a bona fide purchaser protected by the recording statute as to that mortgage. As to the bank’s mortgage, the buyer was a bona fide purchaser because he had no notice of that mortgage executed by the owner, but he did not record until after that mortgage was recorded. Hence, the buyer holds the property subject to both mortgages. The bank’s mortgage is subordinate to the finance company’s mortgage because mortgagees for value are treated as “purchasers” under the recording statutes, and the finance company executed its mortgage without notice of the bank’s prior mortgage and recorded it before the bank recorded its mortgage.
84
Q

Real Property

A seller entered into an enforceable written agreement to sell her house to a buyer for $425,000. The agreement provided that closing would take place on September 18, and on that date the seller would provide marketable title, free and clear of all encumbrances. The agreement was silent as to risk of loss if the house was damaged prior to closing and as to any duty to carry insurance. On August 31, the seller cancelled her homeowners’ insurance when she moved out of the house. Consequently, when the house was destroyed by wildfires on September 15, it was uninsured. The buyer refused to close on September 18 and the seller immediately brought an action against him for specific performance. The buyer countersued for the cancellation of the contract and return of his earnest money. Both parties stipulate that the value of the property without the house is $225,000.

In this jurisdiction, which has no applicable statute, is the seller likely to prevail?

A

Yes, for the full contract price.

  • The seller will most likely prevail for the full contract price. Although jurisdictions differ as to which party has the risk of loss, the majority rule is that where property subject to an enforceable contract for sale is destroyed without the fault of either party before the date set for closing, the risk of loss is on the buyer. Thus, the buyer must pay the contract price despite a loss due to fire, unless the contract provides otherwise. Here, the house was destroyed by fire after the seller and buyer entered into their contract for the sale of the house, but before the closing date. The contract was silent regarding the risk of loss. Thus, under the majority rule, the risk of loss is on the buyer. As a result, the seller is entitled to receive specific performance of the contract, meaning that the buyer must pay the full contract price.
85
Q

Real Property

An owner of three acres of lakefront property subdivided it and sold two acres to a buyer, retaining the one acre actually fronting on the lake. The deed for the two acres expressly included an easement over the westernmost 30 feet of the one-acre parcel retained by the owner for access to the lake. The buyer recorded his deed in the county recorder’s office, which maintained an alphabetical grantor-grantee index only. Fifteen years later, the owner died, leaving the one-acre parcel to his wife. She sold it to a developer that planned to build condominiums. A month later, the buyer died, and his two acres passed by will to his nephew. Three weeks after taking title to the property, the nephew visited the property and discovered that the developer had erected a chain link fence all along the boundary between the nephew’s land and the acre of lakefront land. The nephew brings an action to enjoin the developer from obstructing his easement across the acre of lakefront property.

Which of the following best describes why the nephew should prevail in this litigation?

A

The nephew’s easement is a legal interest that the developer has record notice of, even though there is no tract index.

  • The nephew should prevail because his interest is a legal interest in the property and could have been discovered by the developer in the grantor-grantee index. The owner granted the buyer an easement by express grant. The easement was properly recorded with the buyer’s deed, and because it contained no limitation, it is perpetual. The easement here is appurtenant (i.e., one benefiting the holder of the easement), because it benefits the buyer’s land (the dominant tenement) and burdens the owner’s land (the servient tenement). Where there is an easement appurtenant, it passes with a transfer of the dominant tenement, even though it is an interest in the servient tenement. Thus, the buyer’s easement passed to his nephew. Because the easement is perpetual, it is binding on all of the owner’s subsequent transferees regardless of whether the conveyance refers to the easement, as long as the transferees have notice of it. Many courts will find record or constructive notice here because the nephew’s property is adjacent to the developer’s property, is deeded from a common grantor (the owner), and includes the easement in the original deed from the owner.
86
Q

Real Proeprty

A driver borrowed $75,000 from a bank to purchase a tract of land on which to operate his trucking company, securing the debt with a mortgage on the land. The bank promptly and properly recorded its mortgage. A few years later, the driver financed the installation of a truck wash on the land with a $50,000 loan from a finance company, secured by a mortgage on the land. The finance company promptly and properly recorded its mortgage. The driver subsequently defaulted on the bank’s mortgage, leaving an outstanding balance on the bank’s loan of $60,000. However, the driver continued to make payments to the finance company. The bank brought a foreclosure action, joining the finance company in the proceeding. The jurisdiction provides a statutory right of redemption for lienholders.

Does the finance company have any recourse prior to the foreclosure sale to protect its interest?

A

Yes, the finance company may pay off the bank’s mortgage to preserve its own interest on the land.

  • The finance company may pay off the bank’s mortgage to preserve its own mortgage interest on the land. Because foreclosure will destroy all interests that are junior to the mortgage being foreclosed, the junior mortgagee has the right to pay it off (i.e., redeem it) to avoid being wiped out by its foreclosure. Hence, the finance company may pay off the outstanding balance of the bank’s mortgage and be subrogated to the bank’s rights against the mortgagor. (B) is incorrect because the junior lienholder’s right is not dependent on the statutory right of redemption. A statutory right of redemption, available in about half the states, gives the mortgagor and sometimes junior lienholders a statutory right to redeem for some fixed period after the foreclosure sale has occurred; the amount to be paid is generally the foreclosure sale price, rather than the amount of the original debt. Here, regardless of the statute, the finance company has the right to pay off the bank’s mortgage before the foreclosure sale to protect its interest.
87
Q

A landowner owned two heavily wooded adjoining parcels of land containing a number of lakes. She conveyed the eastern parcel, which contained a hunting resort, consisting of a number of rental cabins, to a neighbor. The deed transferring the parcel also granted to “the neighbor, his heirs and assigns, and to invited guests of the resort all hunting rights and use of the woods on the western parcel for the benefit of the resort.” Subsequently, the neighbor assigned his hunting rights to a hunter.

When the landowner discovered the hunter hunting on her land, she brought an appropriate action to declare his rights void.

If the court rules for the landowner, it will be because the neighbor’s right to hunt on the western parcel is:

A: A profit appurtenant.
Correct answer
B: A profit in gross.
C: An easement in gross.
Incorrect answer
D: A license.

A

A: A profit appurtenant.

  • (A) If the court rules for the landowner, it will be because the neighbor’s right to hunt and fish on the western parcel is a profit appurtenant. A profit is a nonpossessory interest in land that entitles the holder of the profit to enter on the servient tenement and take something off of the land (e.g., minerals, timber, oil, or game). Like an easement, a profit may be appurtenant or in gross. If the profit exists to serve a dominant estate, the profit is appurtenant and can only be transferred along with the dominant estate. Conversely, if the profit does not exist to serve a dominant estate, it is a profit in gross and may be transferred separate and apart from the dominant estate. Here, the neighbor has a profit with respect to the game on the western parcel. Because the profit is “for the benefit of the resort” on the eastern parcel, it is appurtenant rather than in gross because it serves the dominant estate (the eastern parcel). Thus, the neighbor’s assignment of the profit to the hunter is void.
88
Q

Real Property

An owner devised his property by will to a friend “so long as one or more dogs are kept on the property; if dogs are no longer kept on the property, then to the American Society for the Prevention of Cruelty to Animals (ASPCA).” The will also provided that the residuary estate would go to the owner’s niece.

In a jurisdiction that has not modified the common law Rule Against Perpetuities, what are the respective interests in the property on the owner’s death?

A

The friend has a fee simple determinable and the niece has a possibility of reverter.

  • The friend will have a fee simple determinable and the niece will have a possibility of reverter on the owner’s death. A fee simple determinable is an estate that automatically terminates on the happening of a stated event and goes back to the grantor. The interest that is left in a grantor who conveys a fee simple determinable is a possibility of reverter, which arises automatically in the grantor and can be devised by will in almost all jurisdictions. Here, the friend has a fee simple that is subject to automatic termination if dogs are no longer kept on the property. As discussed below, the ASPCA’s interest is stricken because it violates the Rule Against Perpetuities. This leaves a possibility of reverter in the niece as the owner’s residuary devisee.
89
Q

Real Property

A testator executed a will, devising his land “to my son and my daughter, share and share alike.” Shortly thereafter, the daughter died intestate, leaving a child as her only heir. The next year, the testator and his son were involved in a car accident. The testator died immediately. The son died six days later, leaving a will that bequeathed his entire estate to his wife. The jurisdiction has the following statute: “If a devisee, including a devisee of a class gift, who is a grandparent or a lineal descendant of a grandparent of the testator is dead at the time of execution of the will or fails to survive the testator, the issue of such deceased devisee shall take the deceased’s share under the will.”

Who owns the land?

A

The daughter’s child and the son’s wife each own an undivided one-half interest in the land.

  • The daughter’s child and the son’s wife each own an undivided one-half interest in the land. At common law, if a will beneficiary died before the testator, the gift to the beneficiary was void. However, this jurisdiction has an anti-lapse statute, which saves the gift for the predeceasing beneficiary’s descendants if the beneficiary herself is a descendant of the testator. Here, when the daughter died, her one-half interest in the land passed to her child under the anti-lapse statute. When the son died, his one-half interest in the land, to which he was entitled on the father’s death when the father’s will took effect, passed through the son’s estate (not the anti-lapse statute) to his wife. Moreover, a conveyance to two or more persons is presumed to create a tenancy in common rather than a joint tenancy unless an intention to create a right of survivorship is clearly expressed. Each co-tenant has the right to possess all portions of the property; no co-tenant has the right to exclusive possession of any part. Therefore, the daughter’s child and the son’s wife each own an undivided one-half interest in the land as tenants in common.
90
Q

Real Property

An aunt executed and delivered a valid warranty deed conveying her home to her niece as a gift. The niece did not record the deed. Two years later, the aunt was involved in an auto accident. She had allowed her auto insurance to lapse and the other driver’s insurance company obtained a judgment against her for $100,000, which it recorded.

A statute in the jurisdiction provides: “Any judgment properly filed shall, for 10 years from filing, be a lien on the real property then owned or subsequently acquired by any person against whom the judgment is rendered.”

When the aunt died five years later, her will left all of her property to the niece. The insurance company filed a claim in probate against the estate for $100,000. The niece, as executrix, seeks a determination from the probate court that the home is not part of the aunt’s estate, having already been conveyed to the niece.

How should the court rule?

A

The home is not part of the estate and thus is not subject to the claim.

  • The home is not subject to the judgment lien even though the niece never recorded the deed. Under the recording acts, a subsequent mortgagee or bona fide purchaser for value generally prevails over the grantee of a prior unrecorded conveyance. However, most recording statutes do not protect subsequent judgment creditors on the theory that the creditor is not offering consideration at the time its lien is created, and the language of the recording statute generally extends protection to “purchasers,” defined as those obtaining an interest in exchange for consideration. Also, courts generally interpret a judgment lien statute like the one in the question to apply to “any land” actually owned by the judgment debtor rather than any land for which the judgment debtor has record title.
91
Q

Real Property

A tenant entered into a written five-year lease to rent an office from a landlord for $6,000 per year beginning October 1. The lease required that rent in the amount of $500 be paid on or before the first of each month. Two months before the five-year term was up, the tenant received a new lease identical to the one he had already signed, except that the lease term began on the upcoming October 1 and the stated amount of rent per month was $600. The tenant returned the lease to the landlord unsigned, with a letter stating that he did not intend to renew the lease and would be moving out on September 30. The tenant did not move out on September 30. On October 1, the landlord received a check for $500 from the tenant. The notation on the check indicated that it was for the October rent. The landlord deposited the check in her account. She then sent a letter to the tenant stating that he was $100 in arrears in his rent. The tenant did not move out of the office during October, and the landlord did nothing to remove him.

Most courts would hold that the tenant has what type of tenancy?

A

A year-to-year tenancy at $600 per month.

  • The tenant is bound to a year-to-year tenancy at $600 per month, which, absent six months’ notice to terminate, will automatically renew on the following September 30. When a tenant continues in possession after the termination of his right to possession, the landlord may evict him or bind him to a new periodic tenancy. In a commercial lease, if the original lease term was for a year or more, a year-to-year tenancy results. If, prior to the termination of the original tenancy, the landlord notifies the tenant that occupancy after termination will be at an increased rent, the tenant will be held to have acquiesced to the new terms if he does not surrender. A periodic tenancy is automatically renewed until proper notice is given. Proper notice for a year-to-year tenancy is six months’ notice at common law (one month’s notice under the modern Restatement view). In this case, the tenant’s lease expired on September 30. Prior to that expiration, the landlord notified him that any continued tenancy would be at $600 per month. Thus, when the tenant remained in possession after September 30, the landlord could choose to hold him to a year-to-year tenancy (because the original term was five years) at $600 per month. Because the new tenancy is a periodic tenancy, it will be automatically renewed on the following September 30 unless one of the parties gives six months’ notice of termination
92
Q

Real Property

A landowner leased a store to a grocer for a term of five years at $10,000 per year, payable in monthly installments. The lease permitted assignments and subleases. After occupying the premises for two years and paying the rent, the grocer transferred the remaining three years of the term to a florist. The agreement between the parties did not have a specific provision regarding payment of rent, instead just referring to the original lease provisions. The florist occupied the premises for two years but paid rent only for the first year. With one year left on the original lease, the florist transferred her leasehold interest to a barber. The barber occupied the premises for one year but did not pay any rent. The landowner brought an appropriate action against the grocer, the florist, and the barber to recover the rent.

Against whom may the landowner recover?

A

The grocer and the florist jointly and severally for $10,000, and the grocer and the barber jointly and severally for $10,000.

  • The landowner can recover against the grocer and the florist jointly and severally for $10,000 and against the grocer and the barber jointly and severally for $10,000. If a tenant makes a complete transfer of the entire remaining term of his leasehold interest, he has made an assignment. In an assignment, the assignee and the landlord are in privity of estate, and each is liable to the other on all covenants in the lease that run with the land. Because the covenant to pay rent runs with the land, an assignee owes the rent directly to the landlord during the time that she is in privity of estate with the landlord. If the assignee reassigns the leasehold interest, her privity of estate with the landlord ends, and she is not liable for the subsequent assignee’s failure to pay rent. Hence, the florist is liable to the landowner for the one year of rent while she was in privity of estate with her, but not for the year that the barber did not pay rent. The barber is liable for the one year of rent while he was in privity of estate with the landowner. The grocer continues to be liable for rent, even though he is no longer in privity of estate with the landowner, based on the contractual obligation in the lease to pay rent, i.e., on privity of contract grounds. For each of the two years of rent, the two liable parties are jointly and severally liable for the rent. Joint liability means both parties can be liable for the whole. Several liability means that each party is only responsible for his share (i.e., he can seek contribution for the portion that he is not responsible for). (A) is wrong because, as discussed above, the barber also is liable for the final year of rent payments because he was in privity of estate with the landowner.
93
Q

Real Property

A landowner conveyed her 20-acre tract of land to a developer and his heirs, “provided that no multi-family dwellings may be built on the property for a period of 25 years. If such construction is undertaken, the grantor may terminate the conveyance and retake the land.” Two years later the landowner died, leaving her nephew as the sole beneficiary under her will. Shortly thereafter, the nephew discovered that the developer was constructing multi-family dwellings on the land. He promptly brought an ejectment action against the developer. The jurisdiction in which the land is located has a statute providing that all future interests are freely devisable and alienable inter vivos. The common law Rule Against Perpetuities is unmodified by statute.

To whom should the court rule that ownership of the land belongs?

A

The nephew, because the developer began constructing multi-family dwellings on the land and the nephew brought an action for ejectment.

  • The nephew has ownership of the land because he has exercised his power to terminate the developer’s fee simple subject to condition subsequent in the land. A fee simple subject to a condition subsequent is created when the grantor retains the power to terminate the estate of the grantee on the happening of a specified event. On the happening of that event, the estate of the grantee continues until the grantor exercises her power of termination (right of entry) by bringing suit or making reentry. Here, the landowner conveyed a fee simple subject to a restriction against constructing multi-family dwellings, retaining a right of entry. She devised this interest to the nephew in her will, as the statute permitted, and the nephew exercised this right by bringing an ejectment action against the developer after the condition was violated. Hence, the nephew will have ownership of the land.
94
Q

Real Property

A father conveyed his property to his son and daughter “as joint tenants with right of survivorship, but if they ever attempt to sell the property during their lifetimes, a right of first refusal based on the sale price is hereby granted to my sister.” The deed was promptly and properly recorded. Unbeknownst to the son or the sister, the daughter quitclaimed her interest in the property to a purchaser. The following month, the daughter was killed in a snowmobile accident. The purchaser of the daughter’s interest filed a suit for partition of the property. The son filed an appropriate counterclaim for quiet title, asserting that he was the owner of the entire parcel. The sister also filed a counterclaim, asserting that her right of first refusal was valid and that she was prepared to exercise her right to purchase the property for the contract price.

In a jurisdiction in which the Rule Against Perpetuities is unmodified by statute, how should the court rule?

A

For the sister, because she has a valid right of first refusal.

  • The court should rule for the sister because she has a valid right of first refusal that she is entitled to exercise. A joint tenancy is a type of concurrent ownership of a parcel of land that is distinguished primarily by the right of survivorship; i.e., when one joint tenant dies, the property is freed of her concurrent interest and the survivor retains an undivided right in the property that is no longer subject to the interest of the deceased co-tenant. To create a joint tenancy, a grantor must explicitly indicate in the conveyance that the parties are to hold as joint tenants. If one joint tenant conveys her interest, the joint tenancy is severed. The new tenant holds as a tenant in common with the remaining joint tenant, so that there is no longer a right of survivorship. Here, the father, the grantor of the property, expressly indicated in the conveyance that the son and the daughter were to take the property as joint tenants with right of survivorship. When the daughter quitclaimed her interest in the property to the purchaser, the joint tenancy between the daughter and the son was severed (despite the fact that the son did not know of the conveyance to the purchaser). However, the daughter’s conveyance triggered the sister’s right of first refusal. A right of first refusal gives its holder the preemptive right to meet any third party’s offer to purchase real estate. Thus, the sister should have been given the opportunity to meet the purchaser’s offer. Because the purchaser’s interest in the property is no greater than the daughter’s interest, the sister may enforce her right of first refusal against the purchaser.
95
Q

Real Property

A landowner had a contract to sell land to a developer. Before closing on the sale, the developer died. In his will, he left his real property to his son and the residuary estate, including all of his personal property, to his daughter. Just after the developer died, a third party offered to purchase the land from the landowner at a higher price than the contract price. The landowner notified the developer’s son that he was canceling the contract, and would not be conveying the land to the developer’s son. The jurisdiction retains the common law rules regarding exoneration.

What are the rights of the developer’s son?

A

He can demand conveyance of the property and compel the daughter to pay the purchase price.

96
Q

Real Property

A seller owned a large parcel of land. The western half was undeveloped, and the eastern half contained a grove of apple trees. The seller gave a buyer a deed conveying “the western half of the parcel from the western boundary to the grove of apple trees, comprising 220 acres.” It was subsequently determined by survey that the land conveyed to the buyer was in fact 229 acres.

In a dispute between the seller and the buyer as to the mistake, which of the following is most accurate?

A

The deed is valid, and the buyer is the owner of 229 acres.

  • The deed is valid and the buyer owns 229 acres. When there is a mistake or inconsistency in the description of property in the deed, one of the rules of construction is that the physical description takes precedence over the quantity description unless there are grounds for reformation of the deed. Reformation is an equitable action in which the court rewrites the deed to make it conform to the intention of the parties. It is granted when the deed does not express the agreement of the parties due to mutual mistake or a scrivener’s error, and may also be granted when there is a unilateral mistake if misrepresentation is involved. Here, the facts indicate that the seller and the buyer were bargaining for a specific physical location (“the western half of the parcel from the western boundary to the grove of apple trees”) and not for a specific number of acres. Thus, there appear to be no grounds for reformation. A conflict in description does not invalidate a deed.
97
Q

Torts

A landowner who had owned and operated a small airport notified the electric company that he was discontinuing operations and that it should shut down the electrical current that had supplied his communications equipment. The equipment had been surrounded by a fence and signs warning of high voltage. Because the electric company had maintained a transformer next to the landowner’s communications equipment that contained many valuable and reusable parts, it decided to leave the power on to prevent theft until it could schedule removal of the transformer. Three days later, a trespasser who knew that the airport had closed went onto the property looking for something to steal. He could find nothing of value except the transformer. He noticed the signs warning of the high voltage but believed that the power had since been turned off. He scaled the fence with the intent to dismantle the transformer. As soon as he touched the transformer, he was seriously injured by the electric current.

If the trespasser asserts a claim against the electric company to recover damages for his injuries, will he prevail?

A

Yes, because the electric company used unreasonable force to protect its property.

  • The trespasser will prevail against the electric company because it did not have the right to use deadly force to protect its property. As a general rule, one may use reasonable force to prevent the commission of a tort against one’s property. However, force that is likely to cause death or serious bodily harm is not permitted when the invasion is threatening property alone. Furthermore, one may not use indirect deadly force when such force could not lawfully be directly used. Because the trespasser was threatening only the property interest of the electric company, the use of deadly force would not be privileged against him. By leaving the power on to prevent theft, the electric company was using indirect deadly force to defend its property where such force could not lawfully be directly used. Hence, it will be liable to the trespasser for his injuries.
98
Q

Torts

A homeowner who regularly borrowed garden tools from his neighbor went to the neighbor’s house to borrow the neighbor’s leaf blower. The neighbor was not at home, but the leaf blower was in his unlocked garage with his other garden tools, and so the homeowner took it. Unbeknownst to the homeowner, the neighbor had drained the oil from the leaf blower’s motor. The homeowner ran the leaf blower for an hour; the motor was totally destroyed because it had no oil.

The value of the leaf blower at the time that the homeowner took it was $300. An identical new leaf blower costs $500. The cost of repairing the motor is $150. A new motor will cost $250.

If the neighbor sues the homeowner on a theory of conversion and is successful, what damages can he recover?

A

$300, but the homeowner will keep the leaf blower.

  • The neighbor is entitled to $300, but the homeowner will keep the leaf blower. If the plaintiff is successful in a conversion action, the measure of damages is the fair market value of the chattel converted. This value is generally computed as of the time and place of the conversion. The defendant is given title upon satisfaction of the judgment so that, in effect, there is a forced sale of the chattel. (Note that even if the defendant wishes to return the item, the plaintiff is not obligated to take it back once it has been converted.) Here, the value of the leaf blower at the time the homeowner took it was $300, so that is what the neighbor is entitled to.
99
Q

Torts

A bicyclist was riding his bicycle in the street when a negligently driven car struck the bike, knocking the bicyclist off the bike and breaking his right ankle. The driver of the car immediately stopped and went to his assistance. She got him to his feet and was slowly moving him toward the curb when a negligently driven taxicab struck him in the left leg. The bicyclist required surgery on both his right ankle and his left leg.

If the bicyclist sues the driver and the cabbie, which of the following best states his right to recover?

A

He can recover from either the driver or the cabbie for the injury to his left leg and recover from the driver only for the injury to his right ankle.

  • The bicyclist can recover from either party for the left leg injury but only from the driver for the right ankle injury. When two or more tortious acts combine to proximately cause an indivisible injury to a plaintiff, each tortfeasor is jointly and severally liable to the plaintiff for the entire damage incurred. Joint and several liability applies even though each tortfeasor acted entirely independently. However, if the actions are independent, plaintiff’s injury is divisible, and it is possible to identify the portion of injuries caused by each defendant, then each will be liable only for the identifiable portion. Here, the cabbie would not be liable for the injury to the right ankle, because the cabbie did not cause the injury.
100
Q

Torts

A backpacker came upon another hiker who had been bitten by a rattlesnake. The backpacker carried the bitten hiker back to his vehicle and drove him toward the nearest hospital. On the way there, while exceeding the posted speed limit, the backpacker lost control of his vehicle and crashed into a tree by the side of the road. He was uninjured, but the snakebitten hiker’s leg was broken. An ambulance soon arrived and took the hiker to the hospital. The emergency room physician committed malpractice that resulted in the loss of the hiker’s leg. The hiker is now suing the backpacker.

Which of the following is the most likely reason why the backpacker will be held liable for the hiker’s injuries?

A: Having undertaken to rescue the hiker, the backpacker is strictly liable for injuries resulting from the rescue.
B: The emergency room physician’s malpractice is a foreseeable intervening cause that does not relieve the backpacker of liability.
C: The backpacker did not conduct himself as a reasonably prudent person in carrying out the rescue of the hiker.
D: The backpacker committed negligence per se when he exceeded the posted speed limit.

A

C: The backpacker did not conduct himself as a reasonably prudent person in carrying out the rescue of the hiker.

If the backpacker undertakes to rescue the hiker, he must be reasonably prudent in doing so. The general rule in tort law is that no legal duty is imposed on any person to affirmatively act for the benefit of others. However, one who gratuitously acts for the benefit of another is then under a duty to act reasonably. If he acts negligently, he will be liable for damage caused thereby.

101
Q

Torts

A passenger on a commuter train left his seat to go to the lavatory at the front of the car. While he was in the aisle, the car moved across intersecting tracks, causing the car to rock. He stumbled and bumped his knee against the lavatory door, aggravating a preexisting circulation problem in his leg that had been controlled by medication. As a result, he had to have several surgeries to correct the circulation problem.

The passenger brought suit to recover damages against the agency that operated the train system. At the jury trial, the following evidence was presented: The passenger testified as to how he was injured and introduced evidence of his medical expenses. His physician testified that the bump aggravated the circulation problem. The engineer of the train testified that the train had not been exceeding the speed limit for that stretch of track, and the agency introduced a report indicating that a subsequent inspection disclosed no problems with the track. The agency also presented uncontroverted evidence that a person in normal health would not have been injured by the bump. At the close of the evidence, the agency moved for a directed verdict.

How should the court rule on the motion?

A

Grant the motion, because there is no evidence that the agency or its employees operated the train negligently.

  • The court should grant the agency’s motion because the passenger has not established a prima facie case of negligence against the agency. To establish a prima facie case for negligence, a plaintiff must show: (i) a duty of care, (ii) breach of that duty, (iii) actual and proximate cause, and (iv) damages. As a common carrier, the agency may have owed the passenger a high duty of care, and therefore would be liable for slight negligence. However, the passenger has offered no evidence to establish that the agency breached its duty, and res ipsa loquitur is not applicable to these facts because the rocking motion of a train is not the type of event that would occur only as a result of negligence. Because the passenger failed to establish a breach of duty, the court should grant the agency a directed verdict.
102
Q

Torts

An off-duty mall security guard was at a bar with his girlfriend when he got into an argument with another patron. The argument escalated and the guard drew out the gun he had been given at work and shot the patron in the chest, killing him. The survivors of the dead patron brought a wrongful death action against the security agency that hired the guard. At trial, they established that the guard had been required to fill out an application listing references and indicating whether he had any prior convictions for offenses involving violence or use of a weapon, which would disqualify him by law from a position as a security officer. The guard had listed as references some aunts and uncles who had not seen him in some time, and he stated that he had no prior convictions. In fact, the guard had several times been convicted of violent assaults using firearms, and records of these convictions were available in a public database. The agency, however, had not investigated the statements on his application.

Will the survivors of the patron likely prevail in the wrongful death action?

A

Yes, because a reasonable employer would have discovered the guard’s prior convictions.

  • The agency is likely to be liable to the patron’s survivors because it should have known of the guard’s prior convictions. Under the doctrine of respondeat superior, an employer is not vicariously liable for the acts of an employee outside the scope of his employment. However, the employer may be liable for its own negligent selection if it has some reason to be on notice that the actions that resulted in harm were likely to occur. Thus, because the plaintiffs can show that a reasonable employer could have learned of the guard’s violent crimes from public records, the agency likely did not act with reasonable care by not even investigating the guard’s statements on his application (and hiring someone as an armed guard who was barred by law from that type of job), and then giving him a gun that he could carry off duty.
103
Q

Torts

An engineer licensed by the state was the principal design engineer for a wastewater treatment plant’s aeration system. Detailed recommendations for designing aeration systems for this type of plant had been published by a panel of engineers after lengthy study. The engineer fully complied with the recommendations in his design. Nevertheless, the treatment plant’s aeration system suffered a major failure, causing the release of bacteria-laden water into a river that damaged a fish hatchery run by the plaintiff.

If the plaintiff sues the engineer and prevails, what is the likely explanation?

A

The engineer knew of a better design that he could have used that would have prevented the failure.

  • If the plaintiff prevails, it will be because the engineer did not use a better design that would have prevented the failure. A person who is a professional is required to exercise such superior judgment, skill, and knowledge as he actually possesses. If the engineer knew of and could have used a better design for the aeration system, he has breached his duty as a professional. If the better design would have prevented the failure of the aeration system, as choice (A) states, this breach was an actual and proximate cause of the damage suffered by the plaintiff.
104
Q

Torts

A strawberry farmer held his farm open to the public to pick strawberries for a fee. The farmer knew that many patrons would eat as many strawberries out in the field as they would bring home with them, so he advertised that no chemical pesticides or fertilizers were used on his strawberries. The owner of the land adjacent to the farm began operating a soap factory, a use allowed by the zoning code. Flakes of an unavoidable chemical byproduct of the soap-making process would drift over onto the farm whenever the wind was blowing in that direction and settle onto the strawberry plants. The flakes caused no harm to the plants themselves but detracted from the appearance of the strawberries as well as their taste if eaten right off the plant; consequently, the farmer’s business declined. On several occasions, the farmer complained to the factory owner, but the owner did nothing, in part because a visit to the county recorder of deeds office had convinced him that he was the true owner of a large part of the strawberry farm, although in fact it was just a recording error.

Can the farmer recover damages for the harm caused to his business from the factory owner?

A

Yes, because the factory owner intended to conduct the activities that caused the particles to fall on the farmer’s land.

  • The farmer can recover damages for trespass to land because the factory owner intended to conduct the activities that caused the trespass. To establish a prima facie case for trespass to land, plaintiff must prove: (i) an act of physical invasion of plaintiff’s real property by defendant; (ii) intent on defendant’s part to bring about the physical invasion; and (iii) causation. Here, flakes of the chemical byproduct of the factory owner’s soap factory physically invaded the farmer’s property when the wind blew. The factory owner intended to bring about the trespass because, after the farmer had complained, the factory owner knew with substantial certainty that the flakes would continue to fall on the farm whenever the wind was right. Finally, the factory owner’s operation of the soap factory was the cause of the flakes settling on the farmer’s strawberries, completing the prima facie case of trespass to land.
105
Q

Torts

A plaintiff was injured when the steering mechanism of a snowmobile failed. He brought a negligence action against the snowmobile manufacturer. The steering mechanism was designed and manufactured by a component manufacturer; the snowmobile manufacturer merely assembled the snowmobile, branded it, and distributed it directly to retailers.

To prevail against the snowmobile manufacturer, what will the plaintiff need to prove?

A

That the steering mechanism was in a defective condition unreasonably dangerous to users, and the defect could have been discovered and corrected if the component manufacturer had exercised reasonable care in its quality control process.

  • The plaintiff will prevail against the snowmobile manufacturer if the component manufacturer was negligent in not discovering and correcting the defect. To establish a prima facie case of negligence in a products liability case, the plaintiff must show: (i) the existence of a legal duty owed by the defendant to that particular plaintiff; (ii) breach of that duty; (iii) actual and proximate cause; and (iv) damages. The duty of care arises when the defendant acts as a commercial supplier of products. A commercial supplier who assembles a product from components manufactured by others is subject to the same liability as the manufacturer of the defective component. To prove breach of duty, the plaintiff must show (i) negligent conduct by the defendant that leads to (ii) the supplying of a defective product. Here, because the snowmobile manufacturer assembled the snowmobile from component parts, including the steering mechanism manufactured by the component manufacturer, and sold the snowmobile as its own product, it will be liable for the negligence of the component manufacturer.
106
Q

Torts

A testing lab purchased a wind tunnel as a complete unit from a machinery company. The machinery company used an electronics company for the design and installation of the unit’s electronic control systems, which regulated air speed and triggered the emergency shut-off devices.

A technician was installing a scale model of a prototype aircraft that was to be tested in the wind tunnel when the electronic control system of the tunnel malfunctioned, causing the huge fans that created the air flow to start up. The powerful air flow pinned the technician against the grating covering the intake ducts, asphyxiating him before he was discovered and the fans could be shut off.

In an action by the technician’s survivors against the electronics company, proof that the machinery company failed to inspect the wind tunnel has which of the following legal effects?

A

The failure of the machinery company to inspect the tunnel has no legal effect on the electronics company’s liability.

  • While the facts do not indicate that a reasonable inspection by the machinery company would have disclosed any defects, even if an inspection would have disclosed the defect, the machinery company’s failure to do so would have no legal effect on the electronics company’s liability. Regardless of whether the technician’s survivors are using a negligence theory or a strict liability theory (both theories must be considered because the call of the question does not supply the theory of liability), an intermediary’s negligent failure to discover a defect is not a superseding cause, and the defendant who supplied the defective product will be held liable along with the intermediary. Thus, even if the machinery company’s failure to inspect were negligent, it would not relieve the electronics company of liability.
107
Q

Torts

A woman chastised her roommate when she saw that neither the roommate nor her boyfriend wore a helmet when they rode on the boyfriend’s motorcycle. The roommate said that helmets were too restricting. The woman’s brother had died in a motorcycle accident because he had not worn a helmet, so she decided to do something to make a lasting impression on her roommate. She called her roommate at work one day and left a message that the roommate’s boyfriend was in a motorcycle accident and was in the hospital on life support. The roommate was very upset when she got the message and left immediately for the hospital. When she found out later that the message was not true, she became even more upset.

If the roommate brings an action against the woman to recover for her emotional distress, is she likely to prevail?

A

Yes, because the woman knew that there was a high likelihood that the roommate would suffer severe emotional distress.

  • The roommate is likely to prevail because the woman acted in reckless disregard of a high probability that emotional distress would result. To establish a prima facie case for intentional infliction of emotional distress, plaintiff must show: (i) an act by defendant amounting to extreme and outrageous conduct, (ii) intent on the part of defendant to cause plaintiff to suffer severe emotional distress, or recklessness as to the effect of defendant’s conduct, (iii) causation, and (iv) damages-severe emotional distress. Here, the woman’s conduct was extreme and outrageous, and she caused the roommate to suffer severe emotional distress. The facts indicate that she wanted to make a lasting impression on her roommate, so she knew that there was a high likelihood that the roommate would suffer severe distress; hence, she has acted with the mental state of recklessness, and the roommate can establish the prima facie case for this tort.